Ch 4 - A2 Practice Test 2

Pataasin ang iyong marka sa homework at exams ngayon gamit ang Quizwiz!

BASIC MATH SKILLS 27) What is 60% of 90? A) 54 B) 58 C) 60 D) 65

A) 54 Multiply 90 by the decimal equivalent of 60%: 0.60 x 90 = 54

PHYSICS 4) A car, starting from rest, accelerates at 10m/s² for 5 seconds. What is the velocity of the car after 5 seconds? A) 2 m/s B) 5 m/s C) 50 m/s D) The answer cannot be determined from the information given.

C) 50 m/s The velocity of the car is equal to the time multiplied by the acceleration.

BASIC MATH SKILLS 46) Express 9/25 as a decimal. A) 0.09 B) 0.18 C) 0.9 D) 0.36

D) 0.36 There are two simple ways to do this: Divide 9 by 25, or recognize without computing that 9/25 = 36/100

BASIC MATH SKILLS 22) The number 12 is 80% of what number? (Enter numeric value only.) __________________________________

(15) In other words, 0.80x = 12. 12.0.8 = 15

BASIC MATH SKILLS 35) What date in Arabic numerals is Roman numeral MCMLV? (Enter numeric value only.) ______________________________

(1955) Reading from left to right, M = 1,000 C = 900 L = 50 and V = 5 making the date 1955

BASIC MATH SKILLS 49) The nursing staff is currently composed of 20 female & 5 male nurses. Over the next year, the clinic hopes to increase the male nursing staff to equal at least 25% of the total nursing staff. If everyone now working retains his or her job, what is the least number of people the clinic will need to hire? (Enter numeric value only.) ______

(2) The key here is that everyone is retaining his or her job, so you are adding to the total. You can do this by guessing and checking. Right now, there are 5/25 male nurses, or 20% of the nursing staff. If you add just 1 more male nurse, the total will be 6/26 male nurses, or 23% of the nursing staff. Adding another brings you to 7/27 male nurses or just more than 25% of the nursing staff. So adding a minimum of 2 staffers would solve your diversity problem - as long as both were male.

BASIC MATH SKILLS 10) On the lot at Stafford Motors are 7 pickup trucks, 7 two-door automobiles, 28 four-door automobiles, and 14 SUVs. What percentage of the vehicles on the lot are SUVs? (Enter numeric value only. If rounding is necessary, round to the whole number.) __________________________

(25) You are asked to find the percentage of SUVs out of the whole number of vehicles, so first you must add to find the number of vehicles in all: 7 + 7 + 28 + 14 = 56. 14 is ¼ of 56 , so the percentage of SUVs is 25%.

BASIC MATH SKILLS 12) What is the least common denominator of 1/6 and 1/15? (Enter numeric value only.) ____________________

(30) The least common denominator is the least number into which both denominators divide. Finding the common multiples is the easiest way to find this number. Factors of 6 = 6, 12, 18, 24, 30, 36. Multiples of 15 = 15, 30, 45,60,75. The least multiple that both numbers have in common is 30.

BASIC MATH SKILLS 47) 105 is 35% of what number? (Enter numeric value only.) __________________________

(300) In other words. 0.35x = 105 105 ÷ 0.35 = 300

BASIC MATH SKILLS 6) How many milliliters are there in 4 liters? (Enter numeric value only.) _____________________

(4,000) One liter = 1,000 milliliters, so 4 liters = 4,000 milliliters.

BASIC MATH SKILLS 17) Nurse Morgan ordered 6 reams of paper at $8.99 per ream and 10 boxes of envelopes at $3.99 a box. How much did she spend in all? (Enter numeric value only. If rounding is necessary, round to the dollar.) __________________________

(94) $8.99 x 6 = $53.94 $3.99 x 10 = $39.99 Adding the two gives you $53.94 + $39.99 = $93.93 Rounding that to the nearest dollar gives you $94. Notice that in this case (although not in every case), rounding first before you multiply gives you the correct answer: ($9 x 6) + ($4 x 10) = $94

BASIC MATH SKILLS 4) Stu purchased a set of 6 cups & 6 plates at a garage sale. The cups were 25 cents apiece, and the plates were 75 cents apiece. If Stu paid with a $10 bill, how much change was he owed? A) $4 B) $4.50 C) $5 D) $5.50

A) $4 First, determine how much Stu spent for the cups: $.25 x 6 = $1.50 Then find out what he spent for the plates: $.75 x 6 = $4.50. Add those sums: $1.50 + $4.50 = $6.00 Finally, subtract that total from $10.00: $10.00 - $6.00 = $4.00

CHEMISTRY 21) What is the oxidation state of the oxygen atom in the compound NaOH? A) -2 B) -1 C) 0 D) +2

A) -2 With a few rare exceptions, oxygen has an oxidation number of -2 in a compound.

BIOLOGY 17) Hemophilia is a sex-linked trait carried on the X chromosome. In an example of a male with hemophilia & a female carrier, what ratio of the offspring are predicted neither to carry nor to manifest the disease? A) 0 female : 1 male B) 1 female : 1 male C) 1 female : 0 male D) 2 female : 1 male

A) 0 female : 1 male Picture a Punnett square. Reading the chart clockwise from top left, the cross would yield one female carrier, one female hemophiliac, one male hemophiliac, and one normal male who neither carries nor manifests the disease.

CHEMISTRY 16) What is the molarity of a solution containing 0.45 moles of NaCl in 4 liters? A) 0.11 M NaCl B) 0.45 M NaCl C) 01.8 M NaCl D) 8.9 M NaCl

A) 0.11 M NaCl You need simply to divide the number of moles by the number of liters to determine the molarity: 0.45/4 = 0.1125, which rounds to 0.11

BASIC MATH SKILLS 18) Of the 1,525 homes sold by Homestyle Realty last year, Clara sold 244. What percentage of the homes did she sell? A) 16% B) 18% C) 22% D) 24%

A) 16% Solve by dividing Clara's homes by the total & expressing the resulting decimal as a percent. 244 ÷ 1,525 = 0.16, or 16%

BASIC MATH SKILLS 32) How many liters are there in 2,500 milliliters? A) 2.5 liters B) 25 liters C) 250 liters D) 25,000,00 liters

A) 2.5 liters 1,000 milliliters = 1 liter, so 2,500 milliliters = 2.5 liters

PHYSICS 18) A 5-cm candle is placed 20 cm away from a concave mirror with a focal length of 10 cm. What is the image distance of the candle? A) 20 cm B) 40 cm C) 60 cm D) 75 cm

A) 20 cm Look at the relationship between the object distance (d₀), the image distance (dì), the image distance (dì), and the focal length (f). Use the equation: 1/f = 1/d₀ + 1/dì where f = 10 cm and d₀ = 20 cm ⅒ = 1/20 + 1/dì The image distance is 20 cm

PHYSICS 13) Which of the following describes a vector quantity? A) 5 miles per hour due southwest B) 5 miles per hour C) 5 miles D) None of the above

A) 5 miles per hour due southwest Only choice A has both direction and magnitude, making it a vector.

READING COMPREHENSION- Fat for Fitness-(C) 10) What is the meaning of the word MODERATION as used in the second paragraph? A) Adequate amounts B) Intervals of time C) Excess D) Variation

A) Adequate amounts Think about other words you know that look like the word in question. To eat foods in moderation means to eat moderate, sensible amounts of those foods.

CHEMISTRY 19) What form of radiation is composed of electrons traveling at around 16,000 km/sec? A) Alpha radiation B) Beta radiation C) Gamma radiation D) Delta radiation

A) Alpha radiation Alpha particles are ejected from a radioactive substance at about 16,000 km/sec. Beta rays (choice B) are streams of electrons that travel faster, at around 130,000 km/sec.

CHEMISTRY 25) Which of these elements has the greatest atomic mass? A) Au B) Ba C) I D) W

A) Au Gold (Au) has an atomic mass of 196, 96655 amu. Barium (Bb) has an atomic mass of 137.327 amu. Iodine (I) has an atomic mass of 126.90447 amu. Tungsten (W) has an atomic mass of 183.84 amu.

BIOLOGY 23) Which light color would be most effective for growing green plants indoors? A) Blue B) Yellow C) Green D) Orange

A) Blue Of the colors listed, the one best absorbed by chlorophyll is blue. Fluorescent grow lights produce a good deal of blue light.

ANATOMY AND PHYSIOLOGY 21) What is the function of amylase? A) Breaking down starch B) Digesting fat C) Breaking down protein D) Absorbing water

A) Breaking down starch Amylase is present in saliva, where it begins to break down starch into sugar.

BIOLOGY 15) What takes place during anaphase? A) Chromosomes move to opposite ends of the spindle. B) Nuclear membrane & nucleoli disintegrate. C) Chromatids line up at the center of the spindle. D) Nuclear membrane & nucleoli form.

A) Chromosomes move to opposite ends of the spindle. Anaphase is the second step in mitosis. At this stage, the chromatids pull apart & move toward the centrioles at opposite sides of the cell. Once separated, they are called chromosomes. As anaphase ends, a complete set of chromosomes is at each end of the spindle.

CHEMISTRY 2) In the presence of an acid, what color is phenolphthalein? A) Clear B) Blue C) Pink D) Red

A) Clear Phenolphthalein is a common indicator used to determine the pH of a substance. In the presence of an acid, it will be colorless. In the presence of a base, it will turn magenta.

VOCABULARY & GENERAL KNOWLEDGE 50) Select the meaning of the underlined word in the following sentence Apply the gel on the gingival MARGIN around the selected teeth using the blunt-tipped applicator included in the package. A) Edge B) Grease C) Incisor D) Cavity

A) Edge The gingival margin is the edge of the gums.

VOCABULARY & GENERAL KNOWLEDGE 17) What is the best description for the word EFFICACY in the following sentence? The quality, safety, and efficacy of counterfeit medicines are not known. A) Effectiveness B) Significance C) Corollary D) Conclusion

A) Effectiveness EFFECTIVENESS and EFFICACY stem from the same root; both mean "the power to produce intended results."

VOCABULARY & GENERAL KNOWLEDGE 4) Select the meaning of the underlined word in the following sentence. What is the PROGNOSIS for those accident victims? A) Expected outcome B) Treatment option C) Fabrication D) Fabrication

A) Expected outcome The word literally translates as "before knowing." It has to do with predicting or forecasting.

READING COMPREHENSION- Food for Seniors-(C) 15) What is the meaning of the word SUPPLEMENTATION as used in the second paragraph? A) Extra B) Organization C) Attachment D) Revision

A) Extra Look at the word in context. Ordinary diets don't give enough for seniors' needs; they need extra, or supplementation.

VOCABULARY & GENERAL KNOWLEDGE 29) Select the meaning of the underlined word in the following sentence. The patient is lying PRONE next to the monitor. A) Face down B) Immovable C) Face up D) Asleep

A) Face down To lie prone is to lie face down. Its opposite of SUPINE, meaning "face up."

BIOLOGY 4) How do RNA and DNA derive their names? A) From the sugar each contains B) From the structure of their nucleotides C) From the information they transfer D) From the formative processes

A) From the sugar each contains RNA is ribonucleic acid; its sugar is ribose. DNA is deoxyribonucleic acid; its sugar is deoxyribose.

BIOLOGY 13) Which structure is found in protists but not in monerans? A) Golgi apparatus B) Chromosome C) Cytoplasm D) Cell Membrane

A) Golgi apparatus Monerans are prokaryotic, and they do not have vacuoles, centrioles, mitochondria, lysosomes, endoplasmic reticula, nuclear membranes, or Golgi apparatuses.

BIOLOGY 7) Which animal has an open transport system? A) Grasshopper B) Earthworm C) Dolphin D) Chicken

A) Grasshopper Unlike many creatures, grasshoppers & other insects have a system in which blood circulates through blood vessels that open into spaces between their body organs. Carbon Dioxide & oxygen are not exchanged through the blood but instead pass between the cells & the outside of the grasshopper through the tracheas & spiracles.

VOCABULARY & GENERAL KNOWLEDGE 1) Select the meaning of the underlined word in the following sentence. The chemicals in plastic containers may be DELETERIOUS to children's health. A) Harmful B) Deleting C) Delightful D) Inoffensive

A) Harmful From the same root as DELETE, which means "to blot out or destroy," DELETERIOUS means "to Injure."

BIOLOGY 20) Whose efficiency of energy is greater? A) Herbivore B) Carnivore C) Omnivore D) Decomposer

A) Herbivore The greatest energy content is at the base of the energy pyramid: therefore, Producers > primary consumers > secondary consumers > decomposers. An herbivore would be a primary consumer; carnivores & omnivores are more likely to be secondary consumers

BIOLOGY 25) What is the term for the breakdown of glycogen into glucose subunits? A) Hydrolysis B) Reduction C) Metabolism D) Transpiration

A) Hydrolysis In hydrolysis, any class of compound is broken down into subunits through the addition of water.

GRAMMAR 26) Select the word or phrase that is misplaced in the following sentence. In the apartment house, the car with the white roof and new tires belongs to our friends. A) In the apartment house B) with the white roof C) and new tires D) to our friends

A) In the apartment house The car is not in the apartment house, the friends are. Moving the phrase IN THE APARTMENT HOUSE to the end of the sentence would improve the logic of the sentence.

GRAMMAR 25) Which sentence is the clearest? A) In the mailbox there was a note from her boyfriend. B) From her boyfriend, there was a note in the mailbox. C) There was in the mailbox a note from her boyfriend. D) In the mailbox there was from her boyfriend a note.

A) In the mailbox there was a note from her boyfriend. If in doubt, read the sentences aloud & pick the one that is least convoluted.

READING COMPREHENSION- Fat for Fitness-(F) 13) What is a conclusion that a reader could draw from this passage? A) It is advisable to choose unsaturated fats over saturated fats. B) Saturated fats are more calorically dense than unsaturated fats. C) Trans fats are more calorically dense than saturated fats. D) Unsaturated fats are mostly found in animal products.

A) It is advisable to choose unsaturated fats over saturated fats. Caloric density is never mentioned. The reader can conclude that unsaturated fat is healthier than saturated fat, making A the right choice.

BIOLOGY 11) Which gives the order of four taxonomic categories from least to most specific? A) Kingdom, phylum, class, order B) Kingdom, phylum, order, class C) Kingdom, order, phylum, class D) Kingdom, class, order, phylum

A) Kingdom, phylum, class, order Kingdom phylum class order family genus species

GRAMMAR 20) Which of the following is spelled correctly? A) Mechanical B) Mechanicle C) Mecanical D) Machinical

A) Mechanical Although it derives from the same root as MACHINE (choice D), the word MACHINE is not in MECHANICAL.

BIOLOGY 1) Which organelle is the site of energy production? A) Mitochondrion B) Vacuole C) Cell Membrane D) Ribosome

A) Mitochondrion The mitochondrion is the organelle found in eukaryotes that supplies most of the cell's energy by manufacturing ATP

PHYSICS 9) How do you determine the velocity of a wave? A) Multiply the frequency by the wavelength. B) Add the frequency and the wavelength. C) Subtract the wavelength from the frequency. D) Divide the wavelength by the frequency.

A) Multiply the frequency by the wavelength. The velocity of a wave is determined by multiplying its frequency by its wavelength.

BIOLOGY 10) Two organisms live in a relationship from which both benefit. What is this called? A) Mutualism B) Parasitism C) Commensalism D) Competition

A) Mutualism Mutualism is the form of symbiosis through which both species receive mutual benefit.

READING COMPREHENSION- Comparative Life Expectancy... (D) 50) A reader might infer from this passage that _________________________. A) Obesity can be detrimental to a long life B) Car crashes are most frequent in the United States C) The Second Amendment should be overturned D) People in Asia & Africa have the shortest life spans

A) Obesity can be detrimental to a long life Choice B may be true, but you cannot infer it from the passage. Choice C is an opinion that may or may not be the author's there is no support for it in the text. Choice B is contradicted by the reference to Japanese women. Choice A, on the other hand, has support: Obesity is linked in the passage to heart disease & diabetes, which in turn are listed as indicators for low life expectancy.

PHYSICS 10) A 50-kg box of iron fishing weights is balanced at the edge of a table. Peter gives it a push and it falls 2 meters to the floor. Which of the following statements is true? A) Once the box hit the floor, it lost its kinetic and potential energy. B) The box had kinetic energy when it was balanced at the edge of the table. C) The box had both kinetic and potential energy after it fell. D) Once the box hit the floor, it still had kinetic energy.

A) Once the box hit the floor, it lost its kinetic and potential energy. The box had potential energy when it was balanced at the edge of the table -- it could move toward the floor. Once the box hit the floor, it had no potential energy -- and when it came to rest, it had no kinetic energy.

READING COMPREHENSION- A New Vision for PE... (C) 40) What is the meaning of the word INSTILL as used in the second paragraph of the passage? A) Impart B) Propagandize C) Demand D) Crete

A) PROPAGANDIZE (choice B) and DEMAND (choice C) have more negative connotations than the context requires, & CREATE (choice D) does not fit at all. To instill is to fill with something, in this case, information. IMPART (choice A) is a reasonable synonym.

GRAMMAR 10) What punctuation is needed in the following sentence to make it correct? Nurse Henry requisitioned a number of items, including pens, pencils, and paper A) Period B) Question mark C) Comma D) Colon

A) Period The sentence is missing an end mark, which must be a period in this case.

READING COMPREHENSION- A New Use for Acupuncture... (C) 32) What is the meaning of the word VIABLE as used in the last paragraph? A) Reasonable B) Unresolved C) Contemporary D) Consistent

A) Reasonable Acupuncture may now be considered a viable option, meaning that it is practicable, or reasonable.

READING COMPREHENSION- Microbes and Health-(B) ① What is the main idea of the passage? A) Recent discoveries are revealing just how critical microorganisms are to maintaining a properly functioning immune system. B) The human body is host to a wide array of microorganisms C) Bacteriotherapy is being used to reintroduce healthy bacteria into a colon that lacks the capability to defend its elf against pathogenic agents. D) Early tests have shown remarkable potential for the possibilities of bacteriotherapy.

A) Recent discoveries are revealing just how critical microorganisms are to maintaining a properly functioning immune system. Remember that the main idea is the most important theme, one that carries throughout the passage. If someone asked you what this passage was about , you would be most likely to answer "the role of bacteria in the development of the immune system," making A the best answer.

ANATOMY AND PHYSIOLOGY 17) Which hormone controls sleep, mood and appetite? A) Serotonin B) Oxytocin C) Cortisol D) Aldosterone

A) Serotonin Oxytocin (choice B) facilitates birth and breast-feeding. Cortisol (choice C) increases blood sugar and controls some aspects of fat breakdown. Aldosterone (choice D) controls reabsorption of ions and regulates blood pressure. Serotonin (choice A) is a neurotransmitter found in the gut that regulates appetite and sleep as well as some aspects of mood.

VOCABULARY & GENERAL KNOWLEDGE 22) Select the meaning of the underlined word in the following sentence. The interns observed the CONTOUR of the patient's rash. A) Shape B) Answers C) Amount D) Extension

A) Shape CONTOUR means "to go around" and refers to the outline of an object.

GRAMMAR 15) Which sentence is grammatically correct? A) Tapping the beat, Ms. Schuster led the chorus in song. B) Ms. Schuster led the chorus in song tapping the beat. C) Ms. Schuster led the chorus, tapping the beat, in song. D) Tapping the beat, the chorus was led in song by Ms. Schuster.

A) Tapping the beat, Ms. Schuster led the chorus in song. Ms. Schuster is the one tapping the beat, so the modifying phrase should be as close as possible to her name.

READING COMPREHENSION- You're Getting Sleepy... (C) 19) Which of the following statements is an opinion? A) The connection between sleep & weight should be explored. B) Many young children average only eight hours of sleep a night. C) People who sleep less are more likely to be obese. D) Researchers in New Zealand studied almost 600 children.

A) The connection between sleep & weight should be explored. The word SHOULD is a clue that this is what someone thinks or believes. The other statements could be checked or proved.

PHYSICS 1) A kilogram of air is compressed from 1 m³ to 0.5 m³. Which statement is true? A) The density is doubled B) The density is halved C) The mass is doubled D) The mass is halved

A) The density is doubled The mass remains the same, at 1 kg. Because the volume is now halved, the density (mass/volume) is doubled.

GRAMMAR 35) Which sentence is grammatically correct? A) The professor explained the chemical reaction to me. B) The professor explained the chemical reaction to I. C) The professor explained the chemical reaction to myself. D) The professor explained the chemical reaction to mine.

A) The professor explained the chemical reaction to me. The correct pronoun is the object of a preposition (to) and must therefore be an object pronoun.

ANATOMY AND PHYSIOLOGY 13) Which organ system is primarily responsible for storing minerals? A) The skeletal system B) The endocrine system C) The lymphatic system D) The cardiovascular system

A) The skeletal system Bone tissue stores a variety of minerals, from calcium to phosphorus, releasing them into the bloodstream as needed.

READING COMPREHENSION- Killer Cosmetics... (E) 25) What is the author's primary purpose in writing this essay? A) To persuade B) To analyze C) To entertain D) To reflect

A) To persuade The author ends with a statement that makes a plea to the reader. He wants to persuade the reader to be a careful consumer of cosmetics.

VOCABULARY & GENERAL KNOWLEDGE 13) Select the meaning of the underlined word in the following sentence. We will do more tests, as the data so far are INCONCLUSIVE. A) Unsettled B) Convincing C) Unfavorable D) Bewildering

A) Unsettled If results of tests are inconclusive, they do not imply a conclusion. The data are unsettled, or undecided.

VOCABULARY & GENERAL KNOWLEDGE 46) Select the meaning of the underlined word in the following sentence. All VOLATILE liquids should be stored and labeled correctly. A) Unstable B) Abnormal C) Lethal D) Blended

A) Unstable A volatile liquid is one that, due to its chemical instability, can easily explode, catch fire, or emit noxious gases.

ANATOMY AND PHYSIOLOGY 15) Beriberi is a disease caused by lack of ____________________________. A) thiamine B) vitamin C C) niacin D) protein

A) thiamine Thiamine (B₁) deficiency is often found in people whose diet consists largely of polished white rice; in refining the rice, the thiamine-rich husk is removed. Beriberi may affect several systems in the body & may lead to paralysis or death.

VOCABULARY & GENERAL KNOWLEDGE 40) Pulse, temperature, and respiration are known as ___________________. A) vital signs B) vital statistics C) vital force D) vital capacity

A) vital signs The recording of vital signs is typical of hospital rounds or checkups. It is a way to indicate the efficient functioning of the body.

GRAMMAR 39) Which word or phrase fits best in the following sentence? Cathy asked many questions the first time she ___________ at the hospital. A) volunteered B) volunteering C) was volunteered D) volunteer

A) volunteered Notice that the first verb in the sentence is in the past tense, so it makes sense that the second verb would be, too.

READING COMPREHENSION- Lifewings... (D) 28) Which statement could be inferred by the reader from the last paragraph of the passage? A) Procedures that Lifewings Partners recommends are always effective. B) Medical mistakes can happen at even the best hospitals. C) City hospitals know more than others about cutting -edge medicine. D) Medical mistakes will one day be completely eradicated.

B) Medical mistakes can happen at even the best hospitals. The founder suggests in his quote that even cutting-edge hospitals can be dangerous, making choice B the best conclusion.

READING COMPREHENSION- Microbes and Health-(C) ② Which of the following is NOT listed as a detail in the passage? A) Babies are born coated with germs from the birth canal. B) Mice and humans respond to microbes similarly. C) Few people share the same microbes in the same quantities. D) Human bodies contain far more microbes than they do cells.

B) Mice and humans respond to microbes similarly. Details A and C appear in paragraph 2, and detail D is in the opening sentence of paragraph 1. Detail B is never mentioned or implied.

PHYSICS 17) The relationship between acceleration & force is expressed in _____. A) Newton's first law of motion. B) Newton's second law of motion. C) Newton's third law of motion. D) none of Newton's laws of motion.

B) Newton's second law of motion. Newton's second law of motion states that the greater the force applied to an object, the greater is its acceleration.

BIOLOGY 9) Where is the ovary located in a flowering plant? A) Anther B) Pistil C) Stamen D) Calyx

B) Pistil The pistil is the female reproductive organ in a flowering plant.

VOCABULARY & GENERAL KNOWLEDGE 2) What is another word for DEFERENTIALLY? A) Imperfectly B) Respectfully C) Competently D) Energetically

B) Respectfully To defer to someone is to show respect.

GRAMMAR 21) Which sentence is written correctly? A) Since completing the technical course Jasper has had several offers. B) Since completing the technical course, Jasper has had several offers. C) Since completing, the technical course, Jasper has had several offers. D) Since completing the technical course; Jasper has had several offers.

B) Since completing the technical course, Jasper has had several offers. Only the introductory clause requires a comma to separate it from the independent clause it precedes.

PHYSICS 3) A bicycle and a car are both traveling at a rate of 5 m/s. Which statement is true? A) The bicycle has more kinetic energy than the car. B) The bicycle has less kinetic energy than the car. C) Both vehicles have the same amount of kinetic energy. D) Only the car has kinetic energy.

B) The bicycle has less kinetic energy than the car. Kinetic energy is the energy of an object in motion. The greater the mass of that object, the greater its kinetic energy.

GRAMMAR 16) Select the word that makes the following sentence grammatically correct. One out of four doctors _______________________ to this plan. A) subscribe B) subscribes C) subscribing D) are subscribed

B) subscribes The subject of the sentence is ONE, so the verb should be singular.

VOCABULARY & GENERAL KNOWLEDGE 8) A blood vessel that is distended is __________________________. A) overturned B) swollen C) twisted D) overlong

B) swollen TENDERE means "to stretch," as in EXTEND. DIS means "apart," as in DISTANT.

VOCABULARY & GENERAL KNOWLEDGE 42) In the world of pharmaceuticals, a recall policy is a policy involving the _____________________________. A) ability to remember B) the withdrawal of products C) manufacture of cosmetics D) effectiveness of customers

B) the withdrawal of products In a recall, the product in question is pulled from the shelves of retailers.

GRAMMAR 8) Which word is used incorrectly in the following sentence? That is the doctor who's diploma is hung on the partition. A) That B) who's C) hung D) partition

B) who's WHO's means "who is." WHOSE means "belonging to whom."

READING COMPREHENSION- Food for Seniors-(B) 14) What is the main idea of the passage? A) Senior citizens need to increase their calcium & vitamin D intake. B) The food pyramid was revised to reflect the needs of seniors. C) Physical activity is a new addition to the food pyramid. D) The revised food pyramid looks more like a food square.

B) The food pyramid was revised to reflect the needs of seniors. Choices A and C are minor details, and choice D is never suggested. The best choice is B.

ANATOMY AND PHYSIOLOGY 25) Which organ system is primarily responsible for preventing water loss? A) The nervous system B) The integumentary system C) The lymphatic system D) The urinary system

B) The integumentary system The integumentary system (the skin and its appendages) waterproofs the body from outside and guards against excess fluid loss from inside.

READING COMPREHENSION- A New Vision for PE... (E) 42) What can the reader infer about the "sweatiest students" referred to in the last sentence in the passage? A) These students are more overweight than anyone else. B) These students have worked harder during class. C) These students do not need extra credit. D) These students are behind all of their classmates.

B) These students have worked harder during class. Rereading the final paragraph of the passage should prove that choice B is the only logical option here.

VOCABULARY & GENERAL KNOWLEDGE 49) If patients are being counseled, what is happening? A) They are being comforted. B) They are being advised. C) They are being deceived. D) They are being healed.

B) They are being advised. The best synonym for COUNSELING is ADVISING.

READING COMPREHENSION- Microbes and Health-(D) ③ What is the author's primary purpose in writing this essay? A) To persuade B) To inform C) To entertain D) To analyze

B) To inform The author is not trying to convince you of anything, as a persuasive essay might do (choice A). The basic purpose here is to provide information on a topic.

READING COMPREHENSION- Caffeine And Pregnancy... (B) 35) What is the main idea of the passage? A) Coffee carries some obvious health risks for people. B) Two cups of coffee a day may be enough to raise the risk of miscarriage. C) There is a link between miscarriages & morning sickness. D) Miscarriage rates are on the rise internationally.

B) Two cups of coffee a day may be enough to raise the risk of miscarriage. What is the passage mostly about? It's about caffeine in coffee & its link to miscarriage. Choices A & D are too broad, & choice C is not part of the essay.

CHEMISTRY 7) You contain two odorous gases in vials with porous plugs. Gas A has twice the mass of Gas B. Which observation is most likely? A) You will smell Gas A before you smell Gas B B) You will smell Gas B before you smell Gas A C) You will smell Gas A but not Gas B D) You will smell Gas B but not Gas A

B) You will smell Gas B before you smell Gas A A gas with a greater mass effuses less rapidly than a gas with a lesser mass. Therefore, the gas with less mass would be smelled sooner. Since both gases are odorous, and both do effuse eventually, choices C and D are incorrect.

GRAMMAR 49) Select the word that will make the following sentence grammatically correct. The lounge needs a thorough cleaning, _________________ the windows must be washed. A) or B) and C) but D) yet

B) and The word AND indicates addition. Or (choice A) indicates choice, and this is not a situation in which one of the other will do. BUT (choice C) and YET (choice D) indicate contrast, but both actions have to do with cleaning, so no contrast is indicated.

VOCABULARY & GENERAL KNOWLEDGE 47) To vivify someone is to _____________________. A) belittle him B) animate him C) alarm him D) spoil him

B) animate him The root in VIVIFY has to do with life, as in REVIVE or SURVIVAL. To vivify is to give life to, or animate.

GRAMMAR 22) Select the phrase that makes the following sentence grammatically correct. There is a fountain ______________________ the driveway & the main entrance. A) among B) between C) by way of D) in conjunction with

B) between If something is situated in the middle of two other things, it is between them.

VOCABULARY & GENERAL KNOWLEDGE 6) CONCISE is best defined as being _________________________. A) exact B) brief C) accurate D) assembled

B) brief CONCISE comes from a root that means "to cut." Other words with this root include PRECISE, EXCISE, and CIRCUMCISE.

VOCABULARY & GENERAL KNOWLEDGE 37) To censure something is to ________________________. A) inspect it B) denounce it C) admire it D) overlook it

B) denounce it CENSURE, from the same root as CENSOR, refers to the expression of extreme disapproval.

VOCABULARY & GENERAL KNOWLEDGE 36) A treatment that is detrimental is _______________________. A) risk-free B) destructive C) agonizing D) effective

B) destructive If it is detrimental, it causes damage.

GRAMMAR 5) Which word is used incorrectly in the following sentence? The professor was formally an intern in this very hospital back in the 1980s. A) professor B) formally C) intern D) very

B) formally The word that means "at a previous time" is FORMERLY, not FORMALLY.

READING COMPREHENSION- Lifewings... (C) 27) The word WATCHDOG as used in the second paragraph of the passage can best be defined as ____________________________. A) companion B) guard C) manager D) punisher

B) guard Patients are dying from medical errors; the authors suggests that they need someone to guard against this.

GRAMMAR 2) Select the phrase that will make the following sentence grammatically correct. Before I had finished supper, James ______________________. A) is texting me on my cell phone B) had texted me on my cell phone C) texts me on my cell phone D) texting me on my cell phone

B) had texted me on my cell phone The verbs in the sentence must be parallel.

GRAMMAR 32) Which word is used incorrectly in the following sentence? The boys leapt up happy o assist their favorite teacher. A) leapt B) happy C) assist D) their

B) happy The boys leapt up HAPPILY, an adverb, not HAPPY, an adjective.

GRAMMAR 42) Select the word that makes the following sentence grammatically correct. Once of the students ________ completed the course. A) have B) has C) are D) is

B) has The subject is ONE, which is singular. Therefore, the correct verb must agree with a singular subject. IS COMPLETED (choice D) is ungrammatical.

GRAMMAR 7) What word is best to substitute for the underlined words in the following sentence? The nurse left the room to look for the paperwork THE NURSE had misplaced. A) her B) he C) his D) it

B) he The only possible pronoun that completes the sentence is a subject pronoun. It could be SHE or HE.

VOCABULARY & GENERAL KNOWLEDGE 5) If a patient's condition is upgraded, he is __________________________. A) getting worse B) improving in status C) released from the hospital D) approved for surgery

B) improving in status An upgrade is an improvement. Moving from serious to fair condition might be one example.

GRAMMAR 40) Which word is NOT used correctly in the context of the following sentence? Does your grimace infer that you loathed the performance? A) grimace B) infer C) loathed D) performance

B) infer To infer is to deduce, or conclude. A person's grimace cannot do that. A grimace may, however, imply or indicate loathing.

GRAMMAR 23) Which word is NOT spelled correctly in the context of the following sentence? The attending physician lead a small entourage of students and interns. A) attending B) lead C) entourage D) interns

B) lead The past tense of LEAD is LED, and that is the word that should be used here. A present-tense sentence might use the word LEADS; either correction would be permissible.

VOCABULARY & GENERAL KNOWLEDGE 45) HPV is likely to involve the _________________________________. A) skull and spine B) mucous membranes C) kidneys D) heart

B) mucous membranes HPV, or human papillomavirus, causes warts and lesions of the oral, genital, and anal mucous membranes.

BASIC MATH SKILLS 24) At the book sale, Geoff paid 35 cents apiece for 5 paperbacks & $2.50 apiece for 3 hardcover books. He gave the clerk a $10 bill. How much change did he receive? A) $0.50 B) $0.70 C) $1.25 D) $1.75

B) $0.70 The paperback cost $.35 x 5, or $1.75 The hardcover books cost $2.50 x 3, or $7.50 Together, they cost $1.75 + $7.50, or $9.25 Subtract that from $10 to find the change: $10.00 - $9.25 = $ 0.75

BASIC MATH SKILLS 16) Lenore's weekly paycheck is $434.79. Of that, she puts 1/3 aside for her share of rent and utilities and spends $75 on groceries. What is left from her weekly paycheck? A) $219.93 B) $214.86 C) $204.26 D) $192. 76

B) $214.86 First, find 1/3 of $343.79 by dividing the toral by 3 -- $144..93 Subtract that from $434.79 to find what Lenore has left after putting aside her share of the rent: $434.79 - $144.93 = $289.86 Now subtract the amount she spends on groceries: $289.86 - $75 = $214.86

BASIC MATH SKILLS 23) Add and simplify: 2/15 + 1/12 = A) 1/9 B) 13/60 C) 7/15 D) 11/12

B) 13/60 First, find the lowest common denominator. 2/15 x 4/4 = 8/60 1/12 = 5/5 = 5/60 8/60 + 5/60 = 13/60

BASIC MATH SKILLS 25) Convert this military time to regular time: 1310 hours A) 1:10 A.M. B) 1:10 P.M. C) 1:31 A.M. D) 1:31 P.M.

B) 1:10 P.M. Military times greater than 1200 are times after noon.

CHEMISTRY 13) How many electrons are shared in a single covalent bond? A) 1 B) 2 C) 3 D) 4

B) 2 In a single bond, one pair of electrons are shared. In a double bond, two pairs of electrons are shared (choice D).

BASIC MATH SKILLS 41) A team from the highway department can replace 14 streetlights in 7 hours of work. If they work a 30-hour week at this job, in how many weeks will they replace all 120 downtown streetlights? A) 1½ weeks B) 2 weeks C) 2½ weeks D) 3 weeks

B) 2 weeks If they replace 14 in 7 hours, they work at a rate of about 2 per hour. They can do 120 in 60 hours, or two 30-hour weeks.

BASIC MATH SKILLS 33) Express the ratio of 13:60 as a percentage. A) 19.5% B) 22% C) 25.5% D) 31%

B) 22% Percentages are essentially ratios where the second number is 100. Divide 13 by 60 to find a decimal equivalent: 13

BASIC MATH SKILLS 30) What is 35% of 70? A) 17.5 B) 24.5 C) 35 D) 50

B) 24.5 Use the decimal equivalent: 0.35 x 70 = 24.50, or 24.5

BIOLOGY 5) Huntington's disease is carried on the dominant allele. In a situation where two heterozygous parents have the disease, what percentage of their offspring are predicted to be disease-free? A) 0% B) 25% C) 50% D) 100%

B) 25% If the disease is carried on the dominant allele, only the aa combination of recessive genes represents a disease-free offspring. On a Punnett square, that would be one out of four offspring.

BASIC MATH SKILLS 15) Divide and simplify: 3 ½ ÷ 1/10 = A) 3 1/20 B) 3 2/11 C) 3 1/5 D) 3 2/23

B) 3 2/11 Begin by expressing the mixed numbers as improper fractions: 3 ½ = 7/2, and 1 1/10 = 11/10. To divide by a fraction, multiply by its reciprocal. Therefore, 7/2 ÷ 11/10 = 7/2 x 10/11, or 70/22. Now reduce to lowest terms: 70/22 ÷ 2/2 = 35/11. Finally, express 35/11 as a mixed number: 3 2/11.

BASIC MATH SKILLS 2) Eighty percent of the class passed with a 75 or higher. If that percent equaled 24 students, how many students were in the whole class? A) 18 B) 30 C) 36 D) 60

B) 30 Think: 0.80x = 24. Solve: x = 24 ÷ 0.80. The answer is 30

BASIC MATH SKILLS 40) A landscaping plan is drawn on a 1:50 scale. If a deck in the plan measures 12 cm by 10 cm, how large is the deck in real life? A) 12 m by 10 m B) 6 m by 5 m C) 5 m by 2 m D) 4 m by 3 m

B) 6 m by 5 m You can find the answer by setting up the easier of the two possible proportions: 1/50 = 10/x, so x = 500 cm. Since 500 cm = 5 m, you should be able to rule out all of the choices but choice B.

PHYSICS 22) A force of 12 kg stretches a spring 3 cm. How far will the spring stretch given a force of 30 kg? A) 6 cm B) 7.5 cm C) 9 cm D) 10.5 cm

B) 7.5 cm Force equals the extension of the spring times a constant (F = dk). If a force of 12 kg stretches this particular spring 3 cm, then the constant is 4. Apply that same constant to the increased force: 30 = d x 4. The distance of the extension must be 30 ÷ 4, or 7.5 cm.

PHYSICS 20) Which vehicle has the greatest momentum? A) A 9,000-kg railroad car traveling at 3 m/s B) A 2,000-kg automobile traveling at 24 m/s C) A 1,500-kg MINI Coupe traveling at 29 m/s D) A 500-kg glider traveling at 89 m/s

B) A 2,000-kg automobile traveling at 24 m/s Momentum is the product of velocity and mass, and the greatest product of the four presented here is 48,000 kg•m/s.

READING COMPREHENSION- Studying Alzheimer's... (B) 43) What is the main idea of the passage? A) More than 24 million people suffer from some form of dementia. B) A recent study looked at the longevity of dementia sufferers. C) Women with dementia live slightly longer than men do. D) Dementia is a heartbreaking disease that is hard on families.

B) A recent study looked at the longevity of dementia sufferers. Choices A and C are details from the passage. Choice D could be a main idea of a different passage; this one never deals with the cost of dementia to families. The best choice is B.

ANATOMY AND PHYSIOLOGY 10) The human skull contains about how many bones? A) About 5 B) About 30 C) About 60 D) About 210

B) About 30 At birth we have 44 separate bony structures in our skulls, but many of those fuse as we age.

BIOLOGY 8) How does an enzyme work on a chemical reaction that occurs in a substrate? A) An enzyme slows down the chemical reaction. B) An enzyme speeds up the chemical reaction. C) An enzyme has no effect on the chemical reaction. D) An enzyme stops most chemical reactions.

B) An enzyme speeds up the chemical reaction. Enzymes are catalysts that speed up reactions. They do this by lowering the amount of energy needed to begin the reaction.

READING COMPREHENSION- A New Vision for PE... (D) 41) Which is NOT listed as an example of a low-impact sport? A) Martial arts B) Basketball C) Yoga D) Weight lifting

B) Basketball Basketball is mentioned, but not as a low-impact sport.

CHEMISTRY 10) What is the name of the compound CH₃-CH₂-CH₂-CH₃? A) Cyclobutane B) Butane C) Butene D) Butyne

B) Butane The compound has four carbon atoms joined by single bonds, making it a molecule of butane.

READING COMPREHENSION- Fat for Fitness-(B) ⑨ Which statement is NOT a detail from a passage? A) Nearly one-third of your calories should come from fat. B) Canola oil is one source of unsaturated fat. C) Most saturated fats are derived from animal products. D) Trans fatty acids put you at risk for heart disease.

B) Canola oil is one source of unsaturated fat. Canola oil is a source of unsaturated fat, but it is never mentioned in the passage.

BIOLOGY 12) Which molecules contain only carbon, hydrogen, & oxygen? A) Lipids & proteins B) Carbohydrates & lipids C) Proteins & carbohydrates D) Nucleic acids & proteins

B) Carbohydrates & lipids Carbohydrates & lipids are made up of different combinations of carbon, hydrogen, & oxygen. Proteins are made up of amino acids, which center around carbon atoms & always contain hydrogen & oxygen in their simple sugars plus nitrogen & phosphates.

ANATOMY AND PHYSIOLOGY 5) For the average person, what is true about caloric intake? A) It should increase with age after age 25 B) It should decline with age after age 25 C) It should remain constant over a lifetime D) It should decline and then increase with age

B) It should decline with age after age 25 The height of calorie intake for healthy, active people is approximately their mid-20s. After that, calorie intake should decline slightly as energy needs decline.

BIOLOGY 2) What happens to glucose during glycolysis? A) Its energy is entirely lost B) It splits into molecules of pyruvic acid C) It is stored in NADH D) It joins with molecules of citric acid

B) It splits into molecules of pyruvic acid ATP energizes glucose so that it can be broken down, & then the glucose molecules in the cytoplasm of the cell split in half, with each half of the six-carbon glucose becoming a three-carbon substance called pyruvic acid, releasing energy in the process in the form of ATP.

READING COMPREHENSION- Comparative Life Expectancy... (C) 49) Which of the following is NOT listed as a detail in the passage? A) Malta beats the United States in life expectancy. B) Japan's male life expectancy is 80.5 years. C) Heart disease contributes to low life expectancy. D) US infant mortality rates are quite high.

B) Japan's male life expectancy is 80.5 years. Although the longevity of Japanese women is discussed in paragraph 2, the longevity of Japanese men is not.

CHEMISTRY 24) What is the correct formula for potassium chloride? A) NaCl B) KCl C) KCl₂ D) ClK

B) KCl Potassium is K, and chlorine is Cl. Putting them together forms potassium chloride. A chloride is simply a compound of chlorine with another element.

CHEMISTRY 22) Which of these intermolecular forces would have the lowest boiling point? A) Dipole-dipole interaction B) London dispersion force C) Keesom interaction D) Hydrogen bonding

B) London dispersion force Weak intermolecular forces have a low boiling point. Strong intermolecular forces have a high boiling point. London dispersion < dipole-dipole < H-bonding < ion-ion London dispersion is the weakest force of those shown. (A Keesom interaction is one form of dipole-dipole interaction.)

PHYSICS 5) A 10-kg object moving at 5 m/s has an impulse acted on it causing the velocity to change to 15 m/s. What was the impulse that was applied to the object? A) 10 kg•m/s B) 15 kg•m/s C) 20 kg•m/s D) 100 kg•m/s

D) 100 kg•m/s An impulse is a change in momentum. The 10-kg object had a change in its velocity of 10 m/s. Using the equation ◮p = m◮v, change in momentum equals mass times change in velocity, you can see that the answer is 100 kg•m/s.

PHYSICS 8) A 110-volt hair dryer delivers 1,525 watts of power. How many amperes does it draw? A) 167,750 amperes B) 1,635 amperes C) 1,415 amperes D) 13.9 amperes

D) 13.9 amperes Based on the equation P = IV, the current is P/V, or power divided by voltage, Dividing 1,525 watts by 110 volts gives you approximately 13.9 amperes.

BASIC MATH SKILLS 36) Lulu is 5 years younger than her sister, who is 7 years older than their 15 year old cousin. How old is Lulu? A) 27 B) 22 C) 19 D) 17

D) 17 You can do this algebraically or as a guess-and-check. Algebraically, Lulu = L, her sister = S, and their cousin = C L = S-5, and S = C + 7 Since C = 15, S = 22, and L = 17

BASIC MATH SKILLS 14) A recent census of visitors to a popular beach showed that there was a ratio of 6:16 surfers to swimmers. Which of the following is a possible actual number of surfers and swimmers at the beach? A) 60:165 B) 72:210 C) 120:300 D) 210:560

D) 210:560 You must find the ratio that is equivalent to 6:16. 60:160 would be equivalent, making choice A incorrect. 72:192 would be equivalent, making choice B incorrect. 120:320 would be equivalent, making choice C incorrect. The answer is choice D: multiplying both sides of the original ratio by 35 gives you 210:560

BASIC MATH SKILLS 13) Lights out on the base is at 10:30 pm. What would that be in military time? A) 1030 B) 1300 C) 1230 D) 2230

D) 2230 Military time starts with hours before noon. After noon, 1:00 pm is 1300, 2:00 pm is 1400, and so on.

PHYSICS 19) A wave in a rope travels at 12 m/s and has a wavelength of 3.2 m. What is the frequency? A) 38.4 Hz B) 15.2 Hz C) 4.6 Hz D) 3.75 Hz

D) 3.75 Hz Speed equals wavelength times frequency (v = f⋋). In this problem, 12 = 3.2⋋, so ⋋ = 3.75 Hz.

BASIC MATH SKILLS 19) How many centimeters are there in 1 foot? A) 4.72 centimeters B) 10 centimeters C) 25.4 centimeters D) 30.48 centimeters

D) 30.48 centimeters Since 1 inch equals 2.54 centimeters, 12 inches (1 foot) equals 2.54 x 12, or 30.48 centimeters.

BASIC MATH SKILLS 50) How many ounces are there in 4 cups? A) 16 ounces B) 24 ounces C) 28 ounces D) 32 ounces

D) 32 ounces There are 8 ounces in a cup and 32 ounces in 4 cups.

ANATOMY AND PHYSIOLOGY 19) Which of the following is an HDL cholesterol level that might warrant regular testing? A) 70 B) 60 C) 50 D) 40

D) 40 HDL ("good") cholesterol levels in healthy adults range from around 40 to 120 mg/dL, with the mean being a bit over 53. Smoking, obesity, and a sedentary lifestyle may lead to lower HDL. A level of 40 would indicate the need for regular monitoring.

PHYSICS 7) A box is moved by a 15 N force over a distance of 3 m. What is the amount of work that has been done? A) 5 W B) 5 N•m C) 45 W D) 45 N•m

D) 45 N•m Work is the force that is applied to an object over a distance: W=Fd. The force is 15 N, andn the distance is 3 m, giving a total amount of work equal to 45 N•m.

PHYSICS 15) Two objects attract each other with a gravitational force of 12 units. If you double the mass of both objects, what is the new force of attraction between the two? A) 3 units B) 6 units C) 24 units D) 48 units

D) 48 units If the mass of each object is double, the force of attraction is quadrupled.

BASIC MATH SKILLS 34) If blanc CDs cost 36 cents for two, how much does it cost to buy 10 blank CDs? A) $0.90 B) $1.35 C) $1.80 D) $3.60

C) $1.80 If two cost 36 cents, one costs 18 cents, and 10 cost $1.80. This should be easy enough to do in your head.

BASIC MATH SKILLS 48) Wallace expects a 3% raise on his salary of $36,055. What will his salary be then? A) $36,375.20 B) $37,095 C) $37,136.65 D) $38, 010.25

C) $37,136.65 Find 3% of $36,055 and add that to Wallace's salary. 0.03 x $36,055 = $1,081.65. $36,055 + $1,081.65 = $37,136.65

BASIC MATH SKILLS 37) Multiply: 0.05 x 0.22 = A) 1.1 B) 0.11 C) 0.011 D) 0.0011

C) 0.011 Ordinarily, multiplying two numbers with two digits right of the decimal point would result in a product with four digits to the right of the decimal point. Here, however, the last digit, zero, is dropped off.

BASIC MATH SKILLS 8) Multiply: 0.6 x 0.55 = A) 0.0033 B) 0.033 C) 0.33 D) 3.3

C) 0.33 Multiplying a numbers with two digits to the right or the decimal point by a number with one digit to the right of the decimal point should result in a product with three digits to the right of the decimal point. However, since the final digit is 0, you can state the product as 0.33 rather than 0.330

BASIC MATH SKILLS 20) In a scale drawing for a toy rocket, 1 inch = 6 inches. If the rocket is 6 inches tall on the drawing, how tall will it be in reality? A) 1 foot B) 6 feet C) 1 yard D) 2 yards

C) 1 yard If 1 inch is equivalent to 6 inches, then 6 inches is equivalent to 36 inches, or 1 yard.

BASIC MATH SKILLS 28) Divide: 727 ÷ 6 = A) 120 r1 B) 120 r3 C) 121 r1 D) 127 r3

C) 121 r1 The calculation should look like this: ____121__r1___ 31) 727 6 ------- 12 12 ------- 07 6 -------- 1

CHEMISTRY 23) Concentrated HCl has a molarity of 12.0. What volume of concentrated HCl should be used to prepare 500mL of a 3.00 M HCl solution? A) 75 mL B) 100 mL C) 125 mL D) 150 mL

C) 125 mL You can use the molarity/dilution equation to solve this. M₁V₁ = M₂V₂ 3 x 500 = 12 × x 1,500 = 12x x = 125

BASIC MATH SKILLS 21) Add: 7.34 + 3.74 + 4.37 = A) 14.45 B) 14.55 C) 15.45 D) 15.55

C) 15.45 You can eliminate choices A & B here just by rounding each added to the nearest one & estimating the answer.

BASIC MATH SKILLS 26) A gross is equal to 12 dozen. If Lanyard Farms sells 15 gross of eggs a week & packages them in one dozen egg containers, how many containers do they need for a week's worth of eggs? A0 15 B) 150 C) 180 D) 2,160

C) 180 Don't get bogged down in the verbiage here; just do the math. A gross is 12 dozen. The farm sells 15 x 12 dozen eggs a week, or 180 dozen eggs, so 180 containers are needed.

CHEMISTRY 15) What is the correct electron configuration for magnesium? A) 1s²2s² B) 1s²2s²2p⁶ C) 1s²2s²2p⁶3s² D) 1s²2s²2p⁶3s²3p¹

C) 1s²2s²2p⁶3s² The electron distribution of an atom is divided into shells, which in turn contain subshells composed of the orbitals in which the electrons reside. In electron configuration, the symbols 1s, 2s, 2p, and so on, are used to designated subshells, with superscripts indicating the number of electrons in each subshell. There is a maximum number of electrons per subshell. Magnesium has atomic number 12, meaning that it has 12 protons, and in its balanced state, 12 electrons. Looking at the superscripts alone should tell you that only the superscripts in choice C add up to 12.

PHYSICS 23) A 2,000-kg car runs around a track at 10 m/s with a centripetal acceleration of 4 m/s². What is the radius of the track? A) 1,000 m B) 400 m C) 25 m D) 12 m

C) 25 m Acceleration equals the speeds squared divided by the radius (a = v²/R). In this case, a = 4 m/s² and v = 10 m/s, so the radius must be 100 ÷ 4, or 25 m. The mass of the car is irrelevant to the problem.

BASIC MATH SKILLS 29) If Latoya spends 15 minutes every day practicing her flute, how much time does she spend practicing over a period of two weeks? A) 1 hour 45 minutes B) 2 hours 30 minutes C) 3 hours 30 minutes D) 3 hours 45 minutes

C) 3 hours 30 minutes You need to multiply 15 minutes by the number of days in two weeks, 14: 15 x 14 = 210 210 minutes is equal to 3 hours 30 minutes

BASIC MATH SKILLS 31) Express 17/5 as a decimal. A) 0.034 B) 0.34 C) 3.4 D) 34

C) 3.4 Simply divide to find the decimal equivalent: 17÷5 = 3.4 It should be clear from looking at the answer choices that only choice C makes sense.

BASIC MATH SKILLS 43) Subtract and simplify: 4/10 - 1/4 = A) 5/20 B) 3/10 C) 3/20 D) 9/40

C) 3/20 Find the least common denominator and subtract: 8/20 - 5/20 = 3/20

CHEMISTRY 3) Fifty-six kg of a radioactive substance has a half-life of 12 days. How many days will it take the substance to decay naturally to only 7 kg? A) 8 B) 12 C) 36 D) 48

C) 36 For a substance to decay from 56 kg to 7 kg it must undergo 3 half-lives (56→28→14→7). If the half-life of the substance is 12 days, then three half-lives would be 36 days.

BASIC MATH SKILLS 42) Divide 4.5 ÷ 0.9 = A) 0.05 B) 0.5 C) 5 D) 50

C) 5 The answer will be very close to 4.5 divided by 1, so the most reasonable answer (without computation) is choice C.

CHEMISTRY 18) To the nearest whole number, what is the mass of one mole of sodium chloride? A) 36 g/mol B) 43 g/mol C) 58 g/mol D) 72 g/mol

C) 58 g/mol A mole of atoms consists of Avogadro's number of atoms an has a mass in grams numerically equal to the atomic weight of the element. So, one mole of salt, NaCl, has a mass in grams equal to the atomic weight of one atom of sodium plus one atom of chlorine -- about 23 + 35, or 58 g.

CHEMISTRY 14) How many neutrons are in an atom of carbon-12? A) 2 B) 4 C) 6 D) 24

C) 6 Calculate the number of neutrons by subtracting the atomic number from the mass number. The mass number of carbon-12 is 12, and the atomic number for carbon is 6, so 12-6=6

CHEMISTRY 11) When balanced, the reaction Fe + O₂→Fe₂O₃ will be A) 2Fe + 2O₂ → 3F₂O₃ B) 4Fe + 6O₂ → 6F₂O₃ C) 2Fe + 3O₂ → 2F₂O₃ D) 4Fe + 3O₂ → 2F₂O₃

D) 4Fe + 3O₂ → 2F₂O₃ In the original equation, there is one atom of iron (Fe) on the left side and two on the right, so the coefficient 2 is needed to balance iron. Before you limit your choices to A and C, look at the oxygen. There are two atoms of oxygen on the left and three on the right. The only way to balance this is by placing a 3 as a coefficient on the left an d2 as a coefficient on the right. That brings the atoms of iron up to four on the right, so a coefficient o f4 on the left is needed to balance.

READING COMPREHENSION- MRSA-(B) ⑤ As used in the second paragraph, what does the term UBIQUITOUS. A) Indigenous B) Often ignored C) Found everywhere D) Dangerous

C) Found everywhere Staphylococcus aureus is ubiquitous in hospitals -- it is found everywhere.

VOCABULARY & GENERAL KNOWLEDGE 25) VENTRAL refers to which part of the body? A) Top B) Bottom C) Front D) Back

C) Front The ventral part of the body is that where the belly is located; VENTER means "belly." In humans, that part is in front.

BASIC MATH SKILLS 3) If a party planner assumes 2 bottles of sparkling water per 5 guests, how many bottles must she purchase for a party of 145? A) 27 B) 36 C) 49 D) 58

D) 58 Set this up as a proportion: 2/5 = x/145. You may cross-multiply to solve: 2 x 145 = 5x. 290 = 5x x = 290/5, or 58

PHYSICS 16) Four 1.5 V batteries are connected in a series. What is the total voltage of the circuit? A) 1.5 V B) 3.0 V C) 4.5 V D) 6.0 V

D) 6.0 V Attaching batteries in a series gives you a voltage equivalent to the combined voltage of all the batteries.

BASIC MATH SKILLS 39) If the outside temperature is currently 22 degrees on the Celsius scale, what is the approximate temperature on the Fahrenheit scale? A) 56°F B) 62°F C) 66.5°F D) 71.6°F

D) 71.6°F The formula is C x 9/5 + 32 = F 22 x 9/5 + 32 = 71.6

BIOLOGY 18) Which is NOT a product of the Krebs cycle? A) ATP B) Carbon dioxide C) Glucose D) NADH

C) Glucose The Krebs, or citric acid cycle, is the third step in carbohydrate catabolism, the breakdown of sugar. Before it begins, glucose has been broken down via glycolysis into pyruvic acid. In the Krebs cycle, pyruvic acid breaks down into carbon dioxide, releasing two ATPs and 10 NADHs.

VOCABULARY & GENERAL KNOWLEDGE 44) What is the best description of an AUDIT CHECK? A) A restraint on appraisals B) A payment for inventory C) A test of hearing D) A review or inspection

D) A review or inspection An audit is literally a hearing, but an audit check has nothing to do with the ears (choice C). It is an inspection, especially of financial records.

GRAMMAR 1) Which sentence is written correctly? A) Having finished the exam early Rudy checked it over for errors. B) Having finished the exam early Rudy checked, it over for errors. C) Having finished the exam early, Rudy checked it over for errors. D) Having finished, the exam early, Rudy checked it over for errors.

C) Having finished the exam early, Rudy checked it over for errors. A comma must appear between the introductory phrase HAVING FINISHED THE EXAM EARLY and the independent clause RUDY CHECKED IT OVER FOR ERRORS. No other commas are needed.

GRAMMAR 4) Which sentence is grammatically correct? A) The geese were flying south in a V formation with hikers on the mountain spotting them. B) As the geese flew south, hikers were on the mountain, spotting the geese in a V formation. C) Hikers on the mountain spotted geese flying south in a V formation. D) In a V formation, geese flying south spotted hikers on the mountain.

C) Hikers on the mountain spotted geese flying south in a V formation. Reading the choices aloud may help you determine which choice has a logical order of phrases and clauses. The least convoluted sentence is choice C.

READING COMPREHENSION- MRSA-(D) ⑦ Which of the following statements is an opinion? A) Hand-washing stations are one means of combatting MRSA infection. B) Not all Staphylococcus bacteria are resistant to antibiotics. C) Hospitals in the United States should quickly adopt the use of new germ-fighting textiles. D) MRSA infection kills more people in the United States than does HIV/AIDS.

C) Hospitals in the United States should quickly adopt the use of new germ-fighting textiles. A statement of fact can be proved or checked. A statement of opinion is what someone thinks or believes. In this case, statements A, B, and D could proved scientifically, but statement C is simply someone's belief.

PHYSICS 6) Which substance's volume would be most affected by temperature change? A) Liquid nitrogen B) Salt crystals C) Hydrogen gas D) Iron filings

C) Hydrogen gas Increasing the temperature of a substance increases its volume and decreases its density. Such changes are far more evident in gases than in liquids or solids.

VOCABULARY & GENERAL KNOWLEDGE 23) What is the meaning of INCORRIGIBLE? A) Clever B) Potential C) Incurable D) Undistinguished

C) Incurable To be incorrigible is to be uncorrectable. It is often used to describe the behavior of of rambunctious young children.

ANATOMY AND PHYSIOLOGY 12) Which is a secondary defense for the body against pathogens? A) Tears B) Urine C) Inflammation D) Mucus

C) Inflammation Tears (choice A), urine (choice B), and mucus (choice D) are all primary lines of defense, because they work to prevent infection. Inflammation (choice C) is an example of a secondary line of defense, because it kicks in once the body has been infected.

VOCABULARY & GENERAL KNOWLEDGE 43) What is the best description for the phrase MONITOR PROGRESS in the following sentence? The agency will monitor progress of the pilot program, reviewing reports & conducting our own inspections if necessary. A) Watch over modernization B) Take charge of expansion C) Keep an eye on improvement D) Pore over succession

C) Keep an eye on improvement Use the sentence context to choose the best definition.

READING COMPREHENSION- You're Getting Sleepy... (E) 21) What is the main idea of the passage? A) Researchers disagree on the optimum amount of sleep for children. B) Bad sleep habits may result in attention deficit disorders. C) Lack of sleep may have negative effects on weight & behavior. D) Sleep deprivation appears to alter certain hormones in children.

C) Lack of sleep may have negative effects on weight & behavior. Choices A, B, and D may be details, but choice C is the only statement that applies to every paragraph in the passage.

READING COMPREHENSION- Lifewings... (F) 30) What is the main idea of the passage? A) Medical mistakes are made in health centers every day. B) Lifewings Partners is made up of an eclectic mix of people. C) Lifewings Partners is working hard to prevent medical errors. D) Consumers should talk to their doctors about hospital safety.

C) Lifewings Partners is working hard to prevent medical errors. Choices B & D are minor details. Choice A is major, but the passage as a whole is about Lifewings, making choice C a better response.

GRAMMAR 50) Which sentence is grammatically correct? A) Maria planted a pear tree in her orchard of apple trees; it was small. B) The pear tree in Maria's apple orchard was planted when it was small. C) Maria planted a small pear tree among the apple trees in her orchard. D) Having planted a small pear tree, apple trees grew in Maria's orchard.

C) Maria planted a small pear tree among the apple trees in her orchard. Choice C contains all of the information in a logical order.

READING COMPREHENSION- Food for Seniors-(E) 17) Which of the following is NOT mentioned as being important for seniors? A) Vitamin B₁₂ B) Calcium C) Nitrates D) Exercise

C) Nitrates Details A & B are found in paragraph 2, & detail D is found in the last paragraph. The author does not mention nitrates at all, & these compounds would be unlikely to be important for seniors or anyone else.

GRAMMAR 33) Select the sentence that is grammatically correct. A) Nurse Lin asked she and I for our opinion. B) Nurse Lin asked me and her for our opinion. C) Nurse Lin asked her and me for our opinion. D) Nurse Lin asked her and I for our opinion.

C) Nurse Lin asked her and me for our opinion. Because the pronouns follow the verb and receive the action, they must be object pronouns. That eliminates choices A and D. Grammatically, it is correct to name oneself last, making choice C the best answer.

READING COMPREHENSION- A New Use for Acupuncture... (D) 33) Identify the overall tone of the essay. A) Cautionary B) Unconvinced C) Optimistic D) Insensitive

C) Optimistic The mention of the study's conclusions in the last paragraph makes the overall tone one of optimism (choice C) rather than skepticism (choice B) or concern (choice A).

BIOLOGY 6) If bacteria are placed in a strong solution of salt water, they will shrink as water moves out of the bacteria. What is this process called? A) Dehydration synthesis B) Hydrolysis C) Osmosis D) Isotonic Transport

C) Osmosis If cells are placed in a very salty solution, water molecules can diffuse across the cell membrane to move from an area of greater concentration of water molecules (inside the cells) to an area with a lesser concentration of water molecules (the solution of salt water).

VOCABULARY & GENERAL KNOWLEDGE 39) Select the meaning of the underlined word in the following sentence. He made a POST HUMOUS donation to the clinic. A) Generous B) Immense C) Postmortem D) Time-honored

C) Postmortem Essentially "post burial" (HUMUS means "ground"), a posthumous donation is made from the estate of someone who has died.

VOCABULARY & GENERAL KNOWLEDGE 3) What is the meaning of PERUSED? A) Passed judgment on B) Engaged in C) Read thoroughly D) Took notes about

C) Read thoroughly To peruse is to examine in detail. You can peruse a book without taking notes on it (choice D).

VOCABULARY & GENERAL KNOWLEDGE 7) Select the meaning of the underlined word in the following sentence. The night nursing staff is ACCOUNTABLE for those records. A) Receptive B) Notorious C) Responsible D) Preoccupied with

C) Responsible If you are accountable, you can account for your actions.

VOCABULARY & GENERAL KNOWLEDGE 27) Select the meaning of the underlined word in the following sentence. At this point, we are providing mainly PALLIATIVE care. A) Scientific B) Surgical C) Soothing D) Serious

C) Soothing Palliative care is care that lessens pain and gives comfort without actually curing.

CHEMISTRY 6) Among the following elements, which is a nonmetal? A) Mercury B) Magnesium C) Sulfur D) Potassium

C) Sulfur Sulfur is located on the upper right side of the periodic table. It is soft, lacks luster, and does not conduct heat or electricity well, making it a nonmetal.

PHYSICS 11) When the heat of a reaction is negative, which statement is true? A) The products have less energy and are less stable. B) The products have more energy and are more stable. C) The products have less energy and are more stable. D) The products have more energy and are less stable.

C) The products have less energy and are more stable. Exothermic reactions are characterized by negative heat flow, because heat is lost into the environment. Exothermic reactions usually proceed from a higher-energy, less stable reactant to a lower-energy, more stable product.

READING COMPREHENSION- Studying Alzheimer's... (D) 45) What is the author's primary purpose in writing this essay? A) To persuade B) To entertain C) To inform D) To analyze

C) To inform The author is not trying to get the reader to do or believe anything (choice A); he is merely giving information (choice C).

BIOLOGY 22) What is the function of decomposition in the carbon cycle? A) To produce light energy B) To convert carbon to fuel C) To release carbon dioxide D) To store food for consumers

C) To release carbon dioxide In the carbon cycle, carbon passes from the environment through the bodies of living organisms and back into the environment. The dead bodies of organisms are broken down by bacteria and other decomposers, which produce carbon dioxide and release it into the environment.

READING COMPREHENSION- A New Use for Acupuncture... (B) 31) What conclusion is suggested by this study from German? A) So far, acupuncture does not have much credibility as a treatment option. B) Acupuncture can relieve women of all menstrual discomfort. C) NSAIDs are generally more effective than acupuncture for treating cramps. D) Acupuncture is a reasonable treatment choice of cramps.

D) Acupuncture is a reasonable treatment choice of cramps. The passage seems to argue against choice A. A 33% improvement is not a cure of all pain, making B the wrong choice. NSAIDs are one treatment, but there is no comparison of their efficacy with acupuncture, so you cannot conclude choice C. The best choice is D.

GRAMMAR 9) Which of the following words fits best in the following sentence? _______________________ the emergency room was not busy last night, the head nurse still did not let anyone end her shift early. A) Because B) Provided that C) As if D) Although

D) Although Substitute the choices in the blank to find the one that makes sense in context.

ANATOMY AND PHYSIOLOGY 2) In which part of the lungs do nearly all the gaseous exchanges between air and blood take place? A) Pleura B) Trachea C) Bronchioles D) Alveoli

D) Alveoli The alveoli are the tiny air sacs in the lungs where the exchange of oxygen and carbon dioxide takes place.

CHEMISTRY 12) Which two elements are most alike in reactivity? A) He and H B) K and Ar C) Cl and P D) Ba and Mg

D) Ba and Mg Reactivity has to do with the tendency of an element to lose electrons. The least reactive elements are the noble gases, and in general, reactivity increases across the periodic table from left to right. Since magnesium and barium are in the same column of the table, they are similar in reactivity.

BASIC MATH SKILLS 11) In what numeric system does 101 refer to this amount: _____________ ? A) Roman B) Arabic C) Decimal D) Binary

D) Binary There are 5 stars in our Arabic or decimal system of numbers. In the binary system, that would be 101, where 1 = 1 four, 0 = 0 twos, and 1 = 1 one.

VOCABULARY & GENERAL KNOWLEDGE 14) Which word names an infected swelling with liquid inside? A) Blast B) Gash C) Hive D) Boil

D) Boil A boil is an inflamed, pus-filled swelling caused by local infection.

BIOLOGY 16) Which is NOT found in the nucleotide of DNA? A) Simple sugar B) Nitrogen base C) Phosphate group D) Citric acid

D) Citric acid DNA is composed of deoxyribose, a simple sugar; nitrogen bases; and a phosphate group.

BIOLOGY 24) Which bacteria are spherical in shape? A) Clostridia B) Bacilli C) Spirilla D) Cocci

D) Cocci Most bacteria are spherical (choice D), rod-shaped (choice B), or spiral-shaped (choice C)

READING COMPREHENSION- Studying Alzheimer's... (C) 44) What is the meaning of the word ONSET as used in the first paragraph? A) Cure rate B) Incubation period C) Remainder D) Commencement

D) Commencement The onset of dementia is its beginning, or commencement.

ANATOMY AND PHYSIOLOGY 23) Which mineral is important for the formation of red blood cells? A) Selenium B) Calcium C) Magnesium D) Copper

D) Copper Although iron is vital for red blood formation, copper is important for proper iron metabolism. A copper deficiency can quickly lead to anemia.

BIOLOGY 19) A student was asked to count birds in a given location over a 24-hour period. Which count would make her data most valid? A) Count birds at one feeder every 6 hours. B) Count birds at three feeders at noon and 6:00 PM. C) Count birds at one feeder at noon and 6:00 PM. D) Count birds at three feeders every 6 hours.

D) Count birds at three feeders every 6 hours. The more measurements made, the more valid the data are likely to be. Using three sites rather than one allows the researcher to increase observations.

READING COMPREHENSION- Killer Cosmetics... (B) 22) What is the author's attitude toward the use of cosmetics? A) Sympathetic B) Mocking C) Tolerant D) Disapproving

D) Disapproving Word choice can hint at the author's attitude. In this case, FRIGHTENING, HEAVY PRICE, and TOXIC MESS all suggest strong disapproval.

ANATOMY AND PHYSIOLOGY 4) In which organ do muscles push food into the stomach via peristalsis? A) Mouth B) Small intestine C) Epiglottis D) Esophagus

D) Esophagus The esophagus connects the stomach and the oral cavity in the human digestive system.

VOCABULARY & GENERAL KNOWLEDGE 28) What does PLASTIC mean? A) Mechanical B) Durable C) Authentic D) Flexible

D) Flexible Although we tend to think of it as a type of material, in its adjectival form, PLASTIC means "capable of being molded," or "flexible."

CHEMISTRY 4) Which compound contains a polar covalent bond? A) O₂ B) F₂ C) Br₂ D) H₂O

D) H₂O If the elements in a covalent bond have different electronegativity values, the unequal sharing of electrons causes the bond to be polar. H₂O is the only choice with a polar covalent bond.

VOCABULARY & GENERAL KNOWLEDGE 35) Select the meaning of the underlined word in the following sentence. Quick action kept the disease from PERMEATING the general population. A) Conquering B) Wounding C) Deterring D) Infiltrating

D) Infiltrating To permeate is to spread throughout, or to infiltrate.

VOCABULARY & GENERAL KNOWLEDGE 12) Select the meaning of the underlined word in the following sentence. The pain of childbirth may often be ACUTE. A) Escalating B) Emergent C) Eased D) Intense

D) Intense The word is from a root that means "sharp."

ANATOMY AND PHYSIOLOGY 9) Which might be a result of stenosis, or narrowing of a heart valve? A) Abdominal pain B) Blood clots C) Edema (swelling) in organs D) Irregular heartbeat

D) Irregular heartbeat Narrowing of the valves means that blood moves with difficulty out of the heart. Results may include chest pain, edema in the feet or ankles, and irregular heartbeat.

READING COMPREHENSION- Lifewings... (E) 29) Which statement about Lifewings Partners is a fact? A) Everyone should have access to Lifewings. B) The people in Life wings are modern heroes. C) Steve Harden was clever to found Lifewings D) Lifewings is based in Memphis, Tennessee.

D) Lifewings is based in Memphis, Tennessee. A fact can be proved or checked, as the location of the organization could easily be.

ANATOMY AND PHYSIOLOGY 7) How does the integumentary system work with the nervous system? A) The integumentary system removes heat from the neurons in the nervous system. B) The nervous system circulates nutrients outward to the integumentary system. C) Touch input via the integumentary system sends messages to the nervous system. D) Messages from the nervous system affect the color and texture of the skin.

C) Touch input via the integumentary system sends messages to the nervous system. The integumentary system includes the skin, hair, nails, and assorted glands. Receptors embedded in the skin receive information regarding heat, pain, air flow, and so on, which is transported through the nervous system to the brain.

VOCABULARY & GENERAL KNOWLEDGE 15) What is another word for ASYMMETRIC? A) Proportionate B) Impartial C) Uneven D) Magnified

C) Uneven Something that is asymmetric is not symmetrical; it is irregular, or uneven.

ANATOMY AND PHYSIOLOGY 24) The dorsal body cavity is ____________ to the ventral body cavity. A) medial B) deep C) posterior D) anterior

C) posterior The dorsal body cavity contains the spinal column, making it posterior, or toward the back of the body, compared to the ventral body cavity, which contains the structures of the chest and abdomen.

GRAMMAR 27) Select the word or phrase that makes the following sentence grammatically correct. The students promised _____________________ themselves with quiet dignity. A) conduct B) conducting C) to conduct D) to be conducted

C) to conduct Only the infinitive phrase works with the rest of the sentence.

GRAMMAR 38) Which word is used incorrectly in the following sentence? Everybody pretends to do so, but few truly understands the essay. A) pretends B) truly C) understands D) essay

C) understands EVERYBODY is singular, so PRETENDS is the correct verb. FEW is plural, so the correct verb is UNDERSTAND.

READING COMPREHENSION- A New Vision for PE... (B) 39) What would be the best title for this passage? A) "Being a Team in PE" B) "A New Kind of Grade" C) "Learning Martial Arts" D) "PE for School and Life"

D) "PB for School and Life" Choice A is never mentioned, choices B and C are too narrow, and choice D comes close to covering the point of the passage -- that PE is being aimed at life outside of school as well as in the classroom.

BASIC MATH SKILLS 45) Subtract and simplify: 5¼ - 3 5/6 = A) 2 ⅙ B) 1 ½ C) 1 7/24 D) 1 5/12

D) 1 5/12 Express each mixed number as an improper fraction: 2 ¼ - 23/6. Then find the lowest common denominator & restate those fraction: 126/24 - 92/24. Solve, & express as a mixed number in lowest terms: 34/24 = 1 10/24 = 1 5/12

BASIC MATH SKILLS 44) If a helicopter files at about 80 mph, how long will it take to travel 140 miles? A) 1 hour 15 minutes B) 1 hour 20 minutes C) 1 hour 30 minutes D) 1 hour 45 minutes

D) 1 hour 45 minutes Think of this as a proportion: 1 hour/80 miles = x hours/140 Cross-multiply to solve: 140 = 80x; x = 140/80, or 1.75 hours, which equals 1 hour 45 minutes.

BASIC MATH SKILLS 5) Express 1.25 as a fraction in lowest terms. A) 1 1/25 B) 1 ⅖ C) 1 ½ D) 1 ¼

D) 1 ¼ 1.25 equals 1 25/100, or 1 ¼ in lowest terms.

READING COMPREHENSION- MRSA-(C) ⑥ What is the main idea of the passage? A) Methicillin may prove to be the best way to keep MRSA from killing more hospital patients. B) A new textile is the best bet to protect hospital patients against the dangers of MRSA. C) There are many different strains of MRSA, but only one is potentially fatal. D) MRSA is a scourge both in and out of hospitals, but there may be ways to reduce the risk.

D) MRSA is a scourge both in and out of hospitals, but there may be ways to reduce the risk. Choice A is illogical, choice B is close but does not cover all of the information in the passage, and choice C is never suggested. The best overall statement about the passage is D.

BIOLOGY 3) What process do cells in the tip of a plant's root undergo to increase in number? A) Meiosis B) Cytokinesis C) Fractioning D) Mitosis

D) Mitosis Mitosis is cell division that produces new cells with the same number of chromosomes as the original cell. Meiosis (Choice A) is the type of cell division that produces cells with half of the organism's normal chromosome number.

READING COMPREHENSION- Killer Cosmetics... (D) 24) The author describes cosmetics in each of the following cultures EXCEPT ____________________________. A) Greek B) Egyptian C) Roman D) Native American

D) Native American A quick skim of the passage should reveal that choice A, B, and C are mentioned, but D is not.

ANATOMY AND PHYSIOLOGY 18) Which part of the brain is most posterior? A) Frontal lobe B) Parietal lobe C) Temporal lobe D) Occipital lobe

D) Occipital lobe The occipital lobe is farthest back in the skull, posterior to the other parts of the brain.

GRAMMAR 47) Which sentence is the clearest? A) I saw the criminals who were arrested on the TV news. B) I saw the criminals on the TV news who were arrested. C) On the TV news, the criminals who were arrested were seen by me. D) On the TV news, I saw the criminals who were arrested.

D) On the TV news, I saw the criminals who were arrested. Who was watching the TV news? Was it the criminals, or was it the person speaking? Only choice D places the modifying phrase next to the word it modifies.

VOCABULARY & GENERAL KNOWLEDGE 26) What is the best description for the term ANODYNE? A) Emulsified B) Liquefied C) Contaminated D) Painkilling

D) Painkilling ANODYNE translates to "without pain."

GRAMMAR 37) Which of the following is spelled correctly? A) Perseptable B) Perseptible C) Perceptable D) Perceptible

D) Perceptible The word is related to PERCEIVE.

VOCABULARY & GENERAL KNOWLEDGE 11) What is another word for TOXIN? A) Impurity B) Element C) Remedy D) Poison

D) Poison A toxin is any poisonous compound.

READING COMPREHENSION- Food for Seniors-(D) 16) Identify the overall tone of the essay. A) Idealistic B) Theatrical C) Unconcerned D) Pragmatic

D) Pragmatic The passage's tone is the attitude of the author toward the material being discussed. The author sticks to the facts in a serious manner, resulting in a pragmatic, or practical, tone.

VOCABULARY & GENERAL KNOWLEDGE 18) If illegal operators are POSING as licensed pharmacists, what are they doing? A) Modeling B) Inquiring C) Positioning D) Pretending

D) Pretending The illegal operators are pretending to be licensed pharmacists.

VOCABULARY & GENERAL KNOWLEDGE 41) Select the meaning of the underlined word in the following sentence. The children became BELLICOSE after too much time in the moving car. A) Beautiful B) Selfless C) Tolerant D) Quarrelsome

D) Quarrelsome BELLI, as in ANTEBELLUM, means "war." Children who are bellicose exhibit warlike behaviour.

PHYSICS 14) Why are boats more buoyant in salt water than in fresh water? A) Salt decreases the mass of the boats. B) Salt increases the volume of the waters C) Salt affects the density of the boats. D) Salt increases the density of the water.

D) Salt increases the density of the water. Fresh water has a density of around 1,000 kg/m³, whereas that of salt water is around 1,030 kg/m³. The difference in density of the fluids makes objects in salt water buoyant than those in fresh water.

GRAMMAR 19) What punctuation is needed in the following sentence to make it correct? Dr. Levine could not answer all of our questions she promised to research the topic & return with a recommendation the next day. A) Colon B) Question mark C) Comma D) Semicolon

D) Semicolon Placing a semicolon between the two clauses (Dr. Levine could not answer all of our questions & she promised to research the topic & return with a recommendation the next day) makes the sentence correct.

READING COMPREHENSION- Caffeine And Pregnancy... (D) 37) Based on this passage, what can the reader infer about the advice physicians will give their pregnant patients about coffee consumption? A) Almost all of them will advise women to stop drinking any caffeine until after the baby is born. B) The majority will ignore the study altogether & continue to advise caffeine in moderation as before. C) All of them will demand additional research be done before they change what they tell their patients. D) Some will continue to make their normal recommendations about caffeine, while others will be more cautious than before.

D) Some will continue to make their normal recommendations about caffeine, while others will be more cautious than before. There is no support for choices A, B, or C, but the final paragraph of the passage certainly implies that doctors will select different options (choice D).

READING COMPREHENSION- Studying Alzheimer's... (E) 46) Identify the overall tone of the essay. A) Anxious B) Irate C) Indifferent D) Sympathetic

D) Sympathetic Referring to dementia as "heartbreaking" indicates a sympathetic tone.

PHYSICS 2) An airplane travels 500 miles northeast and then, on the return trip, travels 500 miles southwest. Which of the following is true? A) The displacement of the plane is 1,000 miles, and the distance traveled is 0 miles. B) The displacement of the plane is 1,000 miles, and the distance traveled is 1,000 miles. C) The displacement of the plane is 0 miles, and the distance traveled is 0 miles. D) The displacement of the plane is 0 miles, and the distance traveled is 1,000 miles.

D) The displacement of the plane is 0 miles, and the distance traveled is 1,000 miles. The plane traveled a total of 1,000 miles, but because it ended back where it started, the displacement is 0.

ANATOMY AND PHYSIOLOGY 16) How does the endocrine system work with the reproductive system? A) The reproductive system transforms minerals into useful nutrients. B) The endocrine system determines the sex of the embryo. C) The reproductive system controls the growth of secondary sex organs. D) The endocrine system produces chemicals that regulate sexual function.

D) The endocrine system produces chemicals that regulate sexual function. The hormones produced by the endocrine system are critical in sexual development and reproduction.

READING COMPREHENSION- Lifewings... (B) 26) Which of the following is NOT listed as a detail in the passage? A) Which of the following is NOT listed as a detail in the passage? A) Many patients die each year from medical error. B) Lifewings Partners educates patients on consumer safety. C) Steve Harden is the founder of Lifewings Partners. D) The most common medical error is overmedication.

D) The most common medical error is overmedication. Choice A appears in paragraph 2; choices B & C are in paragraph 3. Choice D is never suggested.

READING COMPREHENSION- Caffeine And Pregnancy... (E) 38) Which of the following statements is an opinion? A) People continue to debate the risks of caffeine. B) Doctors studied the link between caffeine & miscarriage. C) Two cups of coffee hold about 200 mg of caffeine. D) To be safe, pregnant women must avoid coffee.

D) To be safe, pregnant women must avoid coffee. Choices A, B, and C could be proved, but so far, choice D cannot, It remains a matter of opinion.

READING COMPREHENSION- Fat for Fitness-(E) 12) What is the author's primary purpose in writing this essay? A) To entertain B) To analyze C) To reflect D) To persuade

D) To persuade Although the passage gives a lot of information, the author is mainly trying to convince the reader to eat a moderate amount of unsaturated fat & to consume little or no trans fat.

VOCABULARY & GENERAL KNOWLEDGE 48) A truculent patient is ____________________. A) exhausted B) irresponsible C) supportive D) argumentative

D) argumentative Truculence is fierceness and is often used to describe speech or writing.

ANATOMY AND PHYSIOLOGY 11) The arteries are part of the ______________________. A) nervous system B) endocrine system C) lymphatic system D) cardiovascular system

D) cardiovascular system The arteries carry blood away from the heart as a key feature of the cardiovascular system.

GRAMMAR 13) Select the word in the following sentence that is NOT used correctly. On departing, the patient complimented her therapist & councilor. A) departing B) complimented C) therapist D) councilor

D) councilor She probably complimented her counselor, or one who gave her counsel, rather than her councilor, or one who serves on a council.

GRAMMAR 34) What word is best to substitute for the underlined words in the following sentence? Mr. Menotti's wife collected MR. MENOTTI'S belongings for the ride home. A) her B) him C) he D) his

D) his The wife did not collect her own belongings (choice A); she collected her husband's (choice D).

GRAMMAR 48) Select the phrase or clause that is misplaced in the following sentence. At a presentation he informed us about the dangers of drugs and alcohol in the auditorium. A) At a presentation B) about the dangers C) of drugs and alcohol D) in the auditorium

D) in the auditorium Look for the phrase that, if moved, would improve the sentence. The danger of drugs and alcohol is not limited to the auditorium; a better sentence would be "At a presentation in the auditorium, he informed us about the dangers of drugs and alcohol."

GRAMMAR 12) Select the word of phrase that makes the following sentence grammatically correct. Kalinda spends part of her day __________________ data from the lab. A) inputs B) input C) having input D) inputting

D) inputting Again, reading this aloud & substituting choices should help you find the one that best fits.

CHEMISTRY 1) The energy required to remove the outermost electron from an atom is called ________________ A) Covalent bonding B) electronegativity C) atomic radius D) ionization energy

D) ionization energy The ionization energy of an atom is the energy required to remove the outermost electron from the atom. This energy is needed to overcome the attraction between the positively charged protons and the negatively charged electrons.

GRAMMAR 28) Select the phrase in the following sentence that is NOT used correctly. Before their hospitalization, the children had rarely eat three meals a day. A) Before their B) children had C) had rarely D) rarely eat

D) rarely eat The verb form that goes with HAD is EATEN, not EAT.

GRAMMAR 14) Select the word that makes the following sentence grammatically correct. Having ______________________ one pill at bedtime, the patient was able to sleep for eight hours. A) took B) take C) taking D) taken

D) taken TAKEN is the verb from that goes with HAVING.

GRAMMAR 45) What word is best to substitute for the underlined words in the following sentence? You can hear THE NURSES' laughter all the way down the hallway. A) his B) hers C) them D) their

D) their NURSES' is a plural possessive noun that must be replaced by a plural possessive pronoun.

VOCABULARY & GENERAL KNOWLEDGE 34) A sore with disintegration of tissue might be called an _______________. A) allergy B) erythema C) alopecia D) ulcer

D) ulcer Ulcers may appear on the skin or on a mucous membrane, whether external or internal.

GRAMMAR 29) Select the word or phrase that makes the following sentence grammatically correct. If Angela attends the seminar tomorrow, she ________________ the employee discount. A) gave B) gives C) will given D) will be given

D) will be given The action takes place tomorrow, and it takes place to Angela, not by Angela. Only choice D fits.

VOCABULARY & GENERAL KNOWLEDGE 32) What is another word for DELUGE ? A) Earthquake B) Dimness C) Downpour D) Chasm

C) Downpour A deluge is a flood, or downpour.

VOCABULARY & GENERAL KNOWLEDGE 31) Another word for the gonads might be the _____________________. A) sperm B) vagina C) testes D) pelvis

C) testes The word GONADS may refer to either male of female sex glands -- either testes or ovaries.

BASIC MATH SKILLS 7) Write the date 2007 in Roman numerals. A) MMVII B) MDVII C) MMDII D) MMXD

A) MMVII 2,000 = MM 7 = VII

BASIC MATH SKILLS 9) Multiply: 0.12 x 0.15 = A) 0.0018 B) 0.018 C) 0.18 D) 1.8

B) 0.018 Multiplying two numbers with two digits to the right of the decimal point should result in a product with four digits to the right of the decimal point. However, in this case, the final digit, 0, is dropped off.

BASIC MATH SKILLS 38) Multiply: 0.6 x 0.06 = A) 0.0036 B) 0.036 C) 0.36 D)3.6

B) 0.036 Since you are multiplying a number with two digits after the decimal point by a number with one digit after the decimal point, the answer should have three digits after the decimal point.

PHYSICS 25) An object with a charge of 4µC is placed 1 meter from another object with a charge of 2 µC. What is the magnitude of the resulting force between the objects? A) 0.04 N B) 0.072 N C) 80 N D) 8 x 10¯⁶ N

B) 0.072 N Use Coulomb's law to find the force. The equation to use is this: F(∊ı∊ⅽ∤) = k x Q₁ x Q₂ / d² Remember that 1 coulomb = 10⁶ microcoulombs. Q₁ this case equals 4 x 10¯⁶ C, Q₂ equals 2 x 10¯⁶ C and d, distance, = 1 m. Now you must recall Coulomb's constant, k = (9.0 x 10⁹ N

BASIC MATH SKILLS 1) Multiply and simplify: 2/3 x 2⅛ = A) 1 7/24 B) 1 5/12 C) 1 11/12 D) 2 1/24

B) 1 5/12 Express 2⅛ as an improper fraction: 17/8. Then multiply numerators & denominators: 2/3 x 17/8 = 34/24 Finally, express this as a mixed number in lowest terms: 1 10/24 = 1 5/12

PHYSICS 24) Two 5-ohm resistors are placed in a series and wired into a 100-V power supply. What current flows through this circuit? A) 2 amp B) 10 amp C) 20 amp D) 50 amp

B) 10 amp According to Ohm's law, current (I) equals voltage (◮V) divided by resistance (R), or I = ◮V/R. In this case, you know voltage and resistance, so I = 100/10, or 10 amp.

READING COMPREHENSION- MRSA-(E) ⑧ Which statement would NOT be inferred by the reader? A) Hospitals may soon limit the spread of infection with a new textile. B) MRSA begins by infecting the skin. C) You are most likely to contract MRSA in a crowded location. D) Used clothing stores may harbor MRSA.

D) Used clothing stores may harbor MRSA. If a statement cannot be inferred by the reader, either there is not enough information to draw conclusions about it, or the passage contradicts the inference. The passage implies that MRSA is transmitted primarily by skin-to-skin contact -- it is described as a "skin infection," and while using someone's razor might put you at risk, buying used clothing that has been washed is unlikely to be hazardous.

GRAMMAR 41) Which sentence is grammatically correct? A) We saw a car parked behind the building with silver tires. B) Behind the building, we saw a car parked with silver tires. C) Parked behind the building, we saw a car with silver tires. D) We saw a car with silver tires parked behind the building.

D) We saw a car with silver tires parked behind the building. To be clear and correct, the phrase WITH SILVER TIRES should lie as close as possible to the word it modifies, CAR, and the phrase PARKED BEHIND THE BUILDING should clearly refer to the car with silver tires, not to the people seeing the car.

READING COMPREHENSION- Caffeine And Pregnancy... (C) 36) What is the meaning of the word OVERTURN in the last paragraph of the passage? A) Justify B) Invalidate C) Support D) Review

B) Invalidate One study is not enough to reverse, or invalidate, the guidelines -- but more studies might do so.

READING COMPREHENSION- Fat for Fitness-(D) 11) Choose the best summary of the passage. A) We often think of all fat as bad, but it is clear that certain fats, especially the more healthful unsaturated fats, are important for bodily function. B) If you must consume some fat, be sure to choose unsaturated fats such as oils rather than saturated fats found in prepackaged snacks or animal products. C) Doctors and researchers have discovered that without the calories found in fat, people would not have functioning metabolisms. D) Some intake of saturated fat is necessary, but you should stay away from the kind of fat found in fried food, which can lead to heart disease.

A) We often think of all fat as bad, but it is clear that certain fats, especially the more healthful unsaturated fats, are important for bodily function. Each choice has something to do with the passage, but only choice A covers the breadth of the topic as it is presented.

CHEMISTRY 20) What is the correct name of ZnSO₄? A) Zinc sulfate B) Zinc sulfide C) Zinc sulfur D) Zinc oxide

A) Zinc sulfate Zn is zinc, and SO₄ is a sulfate. ZnS would be zinc sulfide (choice B). ZnO would be zinc oxide (choice D)

GRAMMAR 6) Select the correct word for the blank in the following sentence. The children and ____________ performed a skit for the elderly patients. A) he B) me C) them D) her

A) he The correct pronoun must be a subject pronoun. Only choice A is a subject pronoun.

VOCABULARY & GENERAL KNOWLEDGE 21) The voice box may be called the _________________________. A) larynx B) lynx C) pharynx D) phalanx

A) larynx A lynx (choice B) is a wildcat; the pharynx (choice C) is part of the alimentary canal, and a phalanx (choice D) may be a bone in the fingers or toes.

GRAMMAR 18) Which word is NOT spelled correctly in the context of the following sentence? The principle role of this clinic has to do with the provision of convenient care. A) principle B) role C) provision D) convenient

A) principle The role of the clinic is principal, meaning "foremost," not principle, meaning "rule".

ANATOMY AND PHYSIOLOGY 14) The vena cavae drain blood from the body into the _________________. A) right atrium B) right ventricle C) left atrium D) left ventricle

A) right atrium Deoxygenated blood first enters the heart via the right atrium.

GRAMMAR 31) Select the phrase that will make the following sentence grammatically correct. Dr. Johnson had a serious look when he ________________________. A) spoke to the patient's parents B) is speaking to the patient's parents C) will speak to the patient's parents D) has spoken to the patient's parents

A) spoke to the patient's parents In this type of question, you must make sure that the tense of verbs remains consistent. Because HAD is past tense, the correct answer contains another past-tense verb, SPOKE.

GRAMMAR 46) Which word is used incorrectly in the following sentence? Moving stealthy, the sneaky little boy reached for the cookie jar on the topmost shelf. A) stealthy B) sneaky C) reached D) topmost

A) stealthy The boy must be moving STEALTHILY; STEALTHY is an adjective, not an adverb.

ANATOMY AND PHYSIOLOGY 6) The corpus callosum facilitates communication between __________. A) the left and right brain B) the skeletal and neural systems C) the brain and spinal cord D) the thalamus and hypothalamus

A) the left and right brain The corpus callosum is a broad band of nerve fibers that join the two hemispheres of the brain.

READING COMPREHENSION- Fat for Fitness-(A)

After years of bad publicity it may sound absurd, but many dietitians are now extolling the virtues of consuming fat. According to Laurie Tansman, a nutritionist at Mt. Sinai Medical Center in New York, fat "not only plays a vital role in the health of the membrane of every cell in our body, it also helps protect us from a number of key health threats." Fat is a critical part of your necessary food intake, and it is generally recommended that 30 % of your daily calories should come from fat. Without it, your body would not be able to absorb or transport vitamins A, D, E, and k. Body temperature, hair follicles, and skin cells are all reliant upon the consumption of fats to function properly. There is near-universal agreement that the healthiest fats are unsaturated fats. This list of healthy fats includes olive oil, sesame oil, walnut oil, avocado oil, and omega-3 fatty acids. These fats, when eaten in moderation, can lower cholesterol levels, thereby reducing your risk for heart disease. Saturated fats are considered less healthy than unsaturated fats & should be consumed more sparingly. Most saturated fats are found in animal products, such as beef, pork & chicken. The least healthy of all fats are trans-fatty acids, or trans fats. These fats can occur in small amounts naturally in animal products, but they are more commonly found as artificially produced oils used for frying foods or softening prepackaged snacks. These fats will increase your risk for heart disease; the U.S. Dietary Guidelines recommend that you consume as few trans fats as possible.

CHEMISTRY 9) Here are the solubilities of four substances at 0°C, in grams of solute per 100 mL of water. Solid citric acid--------------------------------49 Solid potassium phosphate-------------------44 Gaseous nitrogen-----------------------------0.0030 Gaseous oxygen------------------------------0.0070 If the temperature increases to 20°C, what would you expect to happen to the solubility figures? A) Citric acid and potassium phosphate will decrease; nitrogen and oxygen will increase. B) Citric acid and potassium phosphate will increase; nitrogen and oxygen will decrease. C) All four figures will increase. D) All four figures will decrease.

B) Citric acid and potassium phosphate will increase; nitrogen and oxygen will decrease. Substances that are gases at room temperature and pressure decrease in solubility with an increase in temperature. Substances that are solids at room temperature & pressure increase in solubility with an increase in temperature.

GRAMMAR 43) What punctuation is needed in the following sentence to make it correct? The days seem long but the nights seem even longer. A) Period B) Comma C) Colon D) Apostrophe

B) Comma A comma after LONG would separate the two independent clauses.

GRAMMAR 30) What punctuation is needed in the following sentence to make it correct? Because of a major accident on the freeway the emergency room was overcrowded. A) Period B) Comma C) Exclamation point D) Semicolon

B) Comma Placing a comma after FREEWAY separates the introductory adverbial phrase from the rest of the sentence.

CHEMISTRY 5) AB → A + B represents what type of chemical reaction? A) Synthesis B) Decomposition C) Single replacement D) Double replacement

B) Decomposition This type of equation represents decomposition, the breaking down of a compound into its component parts

GRAMMAR 44) Which of the following is spelled correctly? A) Excercise B) Exercise C) Exersize D) Excersise

B) Exercise This is a commonly misspelled word whose spelling must simply be memorized.

READING COMPREHENSION- You're Getting Sleepy... (D) 20) What is the meaning of the word DURATION as used in the last paragraph? A) Basis B) Extent C) Cure D) Solidity

B) Extent The paragraph discusses sleep duration -- the amount of time that people sleep.

READING COMPREHENSION- Killer Cosmetics... (C) 23) What is the meaning of the word CONFORM as used in the first paragraph? A) Authorize B) Fit in C) Hand over D) Bewilder

B) Fit in If you conform to cultural ideals, you fit in with other in your culture.

VOCABULARY & GENERAL KNOWLEDGE 30) What is the best description for the term EUPHORIC? A) Disease-ridden B) High-spirited C) Pretentious D) Understated

B) High-spirited Euphoria is a strong feeling of well-being or ecstasy.

READING COMPREHENSION- Studying Alzheimer's... (F) 47) Choose the best summary of the passage. A) Marital status, age, and gender seem not to be issues in the longevity of patients who suffer from dementia. Out of 438 people of dementia, the vast majority were dead within a few years. B) In hopes of discovering information that can assist with care for dementia patients, Cambridge researchers studied 13,000 seniors. They found that dementia suffers lived on average slightly longer than four years. C) Alzheimer's is just one form of dementia, a medical condition that affects millions and whose prevalence is expected to increase greatly over the next few decades. D) Researchers at Cambridge University have discovered that men with dementia live longer than women with the condition, but frailty is a definite indicator of an early death in patients with dementia.

B) In hopes of discovering information that can assist with care for dementia patients, Cambridge researchers studied 13,000 seniors. They found that dementia suffers lived on average slightly longer than four years. The summary must include all of the main focuses of the passage, and in this case, only choice B fulfills that objective.

READING COMPREHENSION- You're Getting Sleepy... (B) 18) What conclusion can a reader draw about the connection between sleep & young children? A) Lack of sleep causes children to fail in school. B) Inadequate rest raises the risk of behavioral & physical problems. C) Sleeping less than 10 hours a night is guaranteed to result in obesity. D) Eight hours of sleep each night meets the requirements of most children.

B) Inadequate rest raises the risk of behavioral & physical problems. Choice A is a stretch, choice C is even more of a stretch, & choice D is simply untrue, given the information in the final paragraph. The most logical conclusion is B.

VOCABULARY & GENERAL KNOWLEDGE 33) Select the meaning of the underlined word in the following sentence. In children with high fevers, the use of aspirin is CONTRAINDICATED. A) Banned B) Inadvisable C) Anticipated D) Expedient

B) Inadvisable If a treatment is contraindicated, it is not indicated, or inadvisable.

ANATOMY AND PHYSIOLOGY 20) How does a transverse section divide the body? A) Into right and left regions B) Into upper and lower regions C) Into front and back regions D) Between the dorsal and ventral cavities

B) Into upper and lower regions A transverse section is a cross section.

READING COMPREHENSION- Microbes and Health-(E) ④ Choose the best summary of the passage. A) Scientists have used experiments on mice to prove the body's need for certain microorganisms. Instead of damaging the immune system, it turns out that bacteria of certain types may in fact support immunity, at least in lower organisms. B) Bacteriotherapy was originally tried on mice, and the results were positive enough that it now is being used in human trials as well. The essential theory states that introducing bacteria into a damaged colon can help with reconstruction and immune response. C) Although scientists have known for years that the body tolerates certain microorganisms, experiments now show that some bacteria may be required for a healthily functioning immune system. For example, introducing harmless bacteria seems to help the colon defend itself against pathogens. D) Humans are born germ-free, but it does not take long for otherwise healthy immune system. Replacing these harmful bacteria with harmless ones is a new and exciting process known as bacteriotherapy.

C) Although scientists have known for years that the body tolerates certain microorganisms, experiments now show that some bacteria may be required for a healthily functioning immune system. For example, introducing harmless bacteria seems to help the colon defend itself against pathogens. A summary cannot go beyond the information given in the passage, as summaries A, B, and D seem to do. Only summary C focuses on what is suggested by the passage.

GRAMMAR 11) Which of the following is spelled correctly? A) Amature B) Amerture C) Amateur D) Ameteur

C) Amateur From the Latin word AMATOR, or lover, an amateur does something for the love of it, not for money.

GRAMMAR 36) What punctuation is needed in the following sentence to make it correct? In addition to the hospitals regularly scheduled festivities, the custodians celebrate with a party of their own. A) Period B) Comma C) Apostrophe D) Semicolon

C) Apostrophe A comma is used correctly (choice B), and the sentence has an end mark (choice A). It is missing an apostrophe in HOSPITAL'S.

CHEMISTRY 17) Where would you expect tap water to fall on the pH scale? A) Between 1 and 3 B) Between 4 and 6 C) Between 6 and 8 D) Between 8 and 10

C) Between 6 and 8 Tap water is relatively neutral and could be expected to fall in that neutral range.

ANATOMY AND PHYSIOLOGY 8) When the pulmonary value and aortic valves are open, where can blood flow? A) Between the two ventricles of the heart B) From atrium to ventricle within the heart C) Between the heart and the rest of the body D) Between the atria in the heart

C) Between the heart and the rest of the body The two valves connect the heart to the pulmonary artery and the aorta, which carry blood from the heart to the rest of the body.

BIOLOGY 14) How does yeast reproduce? A) Binary fission B) Spore formation C) Budding D) Cloning

C) Budding During budding, a cell or cells in the parent's body produce a smaller version of the parent called a bud. All buds have the exact DNA of the parent. Yeast, hydras, and some plants use this form of asexual reproduction.

ANATOMY AND PHYSIOLOGY 3) How do the intercostal muscles between the ribs assist with respiration? A) By protecting the delicate bronchioles and alveoli B) By signaling a decrease in intra-alveolar pressure C) By enlarging and reducing the space in the thorax D) By maintaining a medial separation between pleurae

C) By enlarging and reducing the space in the thorax The ribs themselves protect the lungs (choice A), but the intercostals are muscles that expand and contract the chest, allowing it to draw in and expel air.

VOCABULARY & GENERAL KNOWLEDGE 20) Select the meaning of the underlined word in the following sentence. It is clear, yellowish liquid having a peculiar ETHEREAL, fruity odor. A) Aerated B) Incapacitating C) Delicate D) Distinctive

C) Delicate Something ethereal is light and airy, like the ether, or upper regions of space.

READING COMPREHENSION- Comparative Life Expectancy... (B) 48) What is the overall tone of the essay? A) Apathetic B) Fascinated C) Discouraged D) Hopeful

C) Discouraged The author's attitude toward the low life expectancy of US citizens seems to be negative. The final paragraph does not offer much hope that the situation will turn around, making choice C the best answer.

PHYSICS 21) How might the energy use of an appliance be expressed? A) Power = energy x time B) Time + energy = power C) Energy = power x time D) Energy/power = time

C) Energy = power x time Using this formula, energy is often measured in kilowatt-hours. For example, if you use a 100-watt light bulb 12 hours a day for 30 days, you will have used 100 x 360 watt-hours, or 36 kilowatt-hours.

VOCABULARY & GENERAL KNOWLEDGE 24) Select the meaning of the underlined word in the following sentence. The patient will need to UNDERGO additional testing. A) Fail B) Offer C) Experience D) Succumb

C) Experience To undergo is to go through, or experience.

VOCABULARY & GENERAL KNOWLEDGE 16) What is the meaning of COUNTERFEIT? A) Over-the-counter B) Foreign C) Fake D) Unidentified

C) Fake COUNTERFEIT literally translates as "to make in opposition." A counterfeit watch is fake.

BIOLOGY 21) How should a researcher test the hypothesis that radiation from cell phones is significant enough to raise the temperature of water in a test tube? A) Dial a cell phone that rests beside a test tube of water, let it ring for two minutes, and record the temperature of the water before and after the two-minute interval. B) Dial a cell phone that rests beside a test tube of water; let it ring for two, three, and four minutes; and record the temperature of the water before and after each interval. C) Use three different brands of cell phone; dial each as it rests beside its own test tube of water, let it ring for two minutes, and record the temperature of the water before and after the two-minute interval. D) Use three different brands of cell phone, dial each and let one ring for two minutes, one for three minutes, and one for four minutes; record the temperature of the water before and after each interval.

C) Use three different brands of cell phone; dial each as it rests beside its own test tube of water, let it ring for two minutes, and record the temperature of the water before and after the two-minute interval. This choice reduces the possibility that one brand might emit more radiation than another; it also allows for a before-and-after measurement that is parallel for each test.

READING COMPREHENSION- A New Use for Acupuncture... (E) 34) Which detail is NOT given as a side effect of NSAIDs? A) Nausea B) Headaches C) Weight Loss D) Dizziness

C) Wight Loss. The complete list of side effects appears in paragraph 2.

CHEMISTRY 8) The three important allotrophic forms of phosphorus are red, white and _____________________. A) green B) gray C) black D) silver

C) black White phosphorus is made by condensing phosphorus vapor. Red phosphorus is made by heating white phosphorus in a vacuum to around 250°C. Black phosphorus is less common and is made by subjecting phosphorus to extreme pressure or by crystallizing white phosphorus.

GRAMMAR 3) Which word is NOT spelled correctly in the context of the following sentence? Everyone in the fraternity found the four boy's pranks sophomoric. A) Everyone B) fraternity C) boy's D) sophomoric

C) boy's If there are four boys, the possessive noun that refers to them all must be BOYS', with an apostrophe after the plural noun.

VOCABULARY & GENERAL KNOWLEDGE 38) The abbreviation DTs on a patient's chart stands for _________________. A) deep tendon reflex B) descending thorax C) delirium tremens D) dextrose in water

C) delirium tremens The DTs are a product of excessive drinking of alcohol. Characteristic signs include sweating, trembling, and hallucianting.

VOCABULARY & GENERAL KNOWLEDGE 19) One muscle in the shoulder is the ___________________________. A) dendrite B) delphic C) deltoid D) dentate

C) deltoid The large triangular muscle that raises the arm from the side is the deltoid, named for the Greek letter DELTA, which is triangular.

ANATOMY AND PHYSIOLOGY 1) The lateral side of the right knee would be _____________________. A) the kneecap B) closest to the left knee C) farthest from the left knee D) on the underside of the knee

C) farthest from the left knee Something that is lateral is toward the outer side of the body. The side of the right knee farthest from the left knee would fit this description.

VOCABULARY & GENERAL KNOWLEDGE 9) Something that is viscous is thick and sticky. Another word for this might be _______________________. A) fluid B) molten C) gelatinous D) malleable

C) gelatinous Something that is fluid (choice A) is liquid. Something that is molten (choice B) is hot and melted. Something that is malleable (choice D) is flexible. Only choice C is a synonym for VISCOUS.

GRAMMAR 24) Select the word that makes the following sentence grammatically correct. Mary Ann gave ______________________ credit for studying so hard and passing the test. A) oneself B) itself C) herself D) themselves

C) herself The reflexive pronoun must correspond to the subject, MARY ANN. Only HERSELF is appropriate in number & gender.

PHYSICS 12) Longitudinal waves have vibrations that move _____________________. A) at right angles to the direction of the vibrations B) in the direction opposite to that of the wave C) in the same direction as the wave D) in waves and troughs

C) in the same direction as the wave The vibrations in longitudinal waves move in the same direction as the wave.

ANATOMY AND PHYSIOLOGY 22) The spleen is part of the __________________________ A) nervous system B) integumentary system C) lymphatic system D) urinary system

C) lymphatic system The spleen uses lymphocytes and macrophages to filter out bacteria, dead tissue, and foreign matter.

VOCABULARY & GENERAL KNOWLEDGE 10) An overt symptom is _________________________. A) dangerous B) hidden C) obvious D) controlled

C) obvious The opposite of OVERT is COVERT, or hidden (choice B).

GRAMMAR 17) What word is best to substitute for the underlined words in the following sentence? The doctor and I went over THE DOCTOR'S & MY notes. A) his B) their C) our D) mine

C) our The notes belong to the doctor and me; they belong to us. The correct possessive pronoun is OUR.

READING COMPREHENSION- Killer Cosmetics... (A)

In recent years, there have been frightening headlines about harmful ingredients such as mercury & lead in ordinary cosmetics. However, these are hardly the first examples of people paying a heavy price to conform to cultural ideals of beauty. That is a tradition that has been around for centuries. Ancient Egyptians decorated their eyes with malachite (a green ore of copper), galena (a lead sulfide), and kohl (a paste made from soot, fat, and metals such as lead). This may have made them look more beautiful, but it also led to health problems such as insomnia & mental confusion. The ancient Greeks went even further. They applied lead to their entire faces, supposedly to clear their complexions of any blemishes and improve the coloration of the skin. Health problems that resulted ranged from infertility to insanity. The lead ointment whitened their faces -- a sure sign of beauty -- and they then added some red lead to their cheeks for the rosy glow. As if that toxic mess were not enough, they also used hair dyes that contained lead. The Romans adopted these practices, and some historians suspect that lead poisoning was part of what later led to the fall of the Roman Empire. As recently as 2007, lipsticks for sale were found to contain lead, & mascara was found to contain mercury. An additional concern is phthalates, industrial chemicals that can cause birth defects & infertility. They are found in personal care products such as shampoos, lotions, perfume, & deodorants. An old saying states that beauty has a price. Sometimes that price may be much higher than consumers realize. Know what you are putting on your face.

READING COMPREHENSION- A New Vision for PE... (A)

In some schools around the country, physical education classes look a lot different than they did a generation or two ago. Kids are still in motion, stretching, running, lifting, and sweating. But instead of everyone doing the same activity at the same time as a team, they are exercising independently. The are being taught movements & activities that their teachers hope they will incorporate into their lives rather than just perform long enough to get a good grade. By teaching kids the pleasure of exercise, gym teachers hope to instill important lessons about maintaining good health, staying fit, & keeping weight under control. Students can also participate in low-impact sports like yoga, martial arts, and weight lifting. Instead of playing basket ball or baseball, they can focus on more general skills like passing the ball. A growing number of physical education (PE) teachers are also putting more of an emphasis on general nutrition & health. With the continual increase in the number of children who are obese, there is greater pressure to teach students how to stay fit. To do this, gym teachers have to look at new ways to introduce exercise to their students that will not intimidate or overwhelm them but instead intrigue & engage them. One other difference found in some modern gym classes is the grading system. Instead of being graded on their ability to run laps in a set time or make a certain number of baskets, the students are graded simply on the effort they make in the class. Some even get extra credit if they are the sweatiest students in the room!

READING COMPREHENSION- MRSA-(A)

Methicillin-resistant Staphylococcus aureus (MRSA) is a form of the Staphylococcus aureus bacterium that is resistant to antibiotics and as a result is very difficult to treat. MRSA now kills more Americans every year than HIV/AIDS, and the rates of infection are rising. Methicillin is an antibiotic that was introduced in the 1960s as a way of combating the Staphylococcus aureus bacterium that is ubiquitous in hospitals. Within a year, doctors began finding strains of bacteria that had already developed immunity to methicillin. By the 1990s, MRSA had become the leading hospital-acquired skin infection in the United States. At the same time MRSA started appearing outside of hospitals. These were different strains of the bacteria, but just as dangerous, and spreading just as quickly. In the past 15 years, MRSA bacteria have become ubiquitous not only in hospitals, but in gyms, locker rooms, swimming pools, and any other setting where human contact is common. Researchers in Ireland are developing a technology that may significantly halt the spread of the hospital-associated MRSA bacteria. They have developed a textile consisting of nanomaterials 1,000 times smaller than a human hair; these textiles are shown to halt the spread of infection and can be used for linens, drapes, and upholstery in hospitals. The potential for this technology to reduce the instances of hospital-associated MRSA is staggering. You can reduce your risk for community-associated MRSA infection by regularly washing your hands, covering all open wounds with a clean bandage, and not sharing any personal items like razors or towels.

READING COMPREHENSION- You're Getting Sleepy... (A)

Most people get a little grumpy when they do not get enough sleep, but when it comes to children, the problem may be more than just some extra irritation. Lack of sleep may also affect their weight as well as their overall behavior. A study conducted in New Zealand at the University of Auckland and published in the medical journal SLEEP followed almost 600 children from infancy through seven years of age. Researchers observed the children's sleep patterns & found that generally they slept less on the weekends than during the week & even less during the summer months. According to the findings, the children who tended to sleep the least were at greater risk for being overweight &/or experiencing behavioral problems. In fact, those who regularly slept less than nine hours a night were three times more likely than longer sleepers to be obese & to show signs of attention deficit disorder(ADD) and attention deficit hyperactivity disorder (ADHD). These results were based on questionnaires completed by the children's parents & teachers. How does sleep affect weight? The answer to that is still not clear, but experts suspect that chronic sleep deprivation somehow alters the hormones involved in appetite control & metabolism. This is a connection that still needs to be explored to be better understood. How much sleep is enough for a child? Experts recommend that preschoolers get 11 to 13 hours of sleep each night, whereas school-age children should get between 10 & 11 hours per night. Many children average only 8 hours. The study concluded that sleep duration is one risk factor that can be fairly easily altered to prevent future health problems for today's young people.

READING COMPREHENSION- A New Use for Acupuncture... (A)

Over the years, acupuncture has become a more widely accepted type of alternative medicine. It is used for a wide variety of ailments, and if a recent study from Germany is valid, relieving menstrual pain can be added to the continuously growing list. Traditionally nonsteroidal anti-inflammatory drugs (NSAIDs) are the typical treatment for menstrual discomfort. However, as many consumers & physicians are aware, NSAIDs have a number of side effects, including nausea, vomiting, rash, dizziness, headache, and drowsiness. Acupuncture rarely has any kind of side effects other than the occasional stinging sensation when the needle is inserted or a deep ache around it after it is in place. Acupuncture has proven helpful with relieving a number of kinds of pain, so researches at Charite University Medical Center in Berlin wanted to find out how effective it might be in combating cramps & other menstrual discomforts. More than too women were enrolled in the study, & after 3 months & approximately 10 sessions, the women who were treated with acupuncture reported significantly less pain than those in the control group who received no treatment at all. They also reported a 33 % improvement in their symptoms. Because of these findings, the researchers came to the conclusion that "acupuncture should be considered as a viable option in the management of those patients."

READING COMPREHENSION- Food for Seniors-(A)

The food pyramid is a visual representation of how the different food groups can be combined to form a healthy diet. Although it was a vital part of dietary guidelines for years before being replaced by the "MyPlate" model, the pyramid was constantly analyzed and revised as additional study was done in nutritional fields. A few years ago, the pyramid underwent a change regarding the unique dietary needs of seniors. Modifications in the pyramid for older adults included an emphasis on fiber and calcium, as well as on vitamins D and B12. By incorporating these changes, the pyramid indicated that the nutrients found in a person's routine daily consumption typically are not enough for seniors. Seniors need supplementation. As people age, they tend to move less & thus need fewer calories to maintain their weight. Because seniors tend to eat a more limited amount, dietitians urge them to choose wisely. They are urged to eat nutrient-rich meals featuring such food as fruits, vegetables, low-fat dairy products, and high fiber whole grains. The revised pyramid also focused on the importance for older people of ingesting adequate amounts of fluids daily. This helps to ensure proper digestion and prevent dehydration. Finally, the revised pyramid included information on incorporating exercise and other physical activities into the lives of older adults. Suggestions included swimming, walking, or simple yard work. Because recent reports have stated that obesity levels for people older than 70 years of age are climbing, performing some type of regular exercise is more essential than ever.

READING COMPREHENSION- Caffeine And Pregnancy... (A)

The health risks of coffee have long been debated, but a recent study has added another argument against too much coffee consumption. This study looked at the effect of drinking coffee on pregnant women. Conducted by physicians at Kaiser Permanente, the study explored the connection between caffeine & the risk of miscarriage. This study followed more than 1,000 women who became pregnant within a two-year period. The amount of caffeine they drank was logged, as well as which women experienced a miscarriage. The results, as published in the January 2008 issue of the American Journal of Obstetrics and Gynecology, stated that the risk of miscarriage more than doubled in women who consumed 200 mg or more of caffeine per day -- about what is found in 2 cups of coffee. Why does caffeine carry this risk? Researchers are not sure, but they theorize that the caffeine restricts blood flow to the placenta. This, in turn, can harm the developing fetus. Does this mean the physicians will start advising women to quit drinking coffee while pregnant? Yes and no. Some doctors will certainly take this report to heart & encourage their patients to stay away from more than one cup of coffee a day, just as they recommend not drinking alcohol or smoking cigarettes. Others are not so convinced & doubt that this single study is enough to overturn the established guidelines of the American College of Obstetricians & Gynecologists. Instead, they believe that a lot more research needs to be done.

READING COMPREHENSION - Microbes and Health-(A)

There are 10 times more microbes than human cells in the human body. Scientists have long known that the human body is host to a staggering number of microorganisms, but recent information is shedding light on just how pivotal a role these bacteria play in the development of the human immune system. The human body plays host to a wide array of microorganisms that are specially adapted to survive in particular portions of the human body. There is such a great amount of variation in these microorganisms that few people will share the same strains of bacteria in the same quantities. This process begins at birth: a newborn infant emerges from the womb, a germ-free environment, and is immediately coated with germs from its mother's birth canal. These germs immediately begin to breed and colonize the human body that will now be its new host. The most intriguing discovery is not that the body's immune system tolerates these millions of harmless organisms, but that it may rely upon their presence to function properly. Scientists recently found that with laboratory mice that could not produce a particular inflammation-reducing molecule, upon being injected with a particular strain of bacteria that was then allowed to breed, their immune system quickly developed the ability to synthesize that molecule. Simply put, the mice needed the bacteria for their immune systems to function properly. This same basic concept is also being used with humans. A relatively experimental process known as fecal bacteriotherapy is now being used to reintroduce healthy bacteria into a colon that lacks the capability to defend itself against pathogenic agents. Scientists are just beginning to understand the important role that these microorganisms play in the body of a healthy human being, but early tests have yielded remarkable discoveries.

READING COMPREHENSION- Lifewings... (A)

What do pilots, astronauts, physicians, and risk managers have in common? In this case, they are all part of an organization based in Memphis, Tennessee, called Lifewings Partners. This unusual group focuses on finding ways to eliminate mistakes & accidents in medical settings within the United States. Lifewings Partners emphasizes the need for a watchdog in various medical settings. According to the Institute of Health, approximately 98,000 patients die each year in U.S. health care settings due to nothing more than medical error. Examples of medical errors include the man who had the wrong testicle removed in a Los Angeles hospital, a young boy who went in for a typical hernia surgery & ended up with brain damage from the anesthesia, and a hospital in Rhode Island that performed brain surgery on the wrong side of the brain -- three times on three different patients in less than a year. In addition to making internal changes in medical settings by changing procedures & establishing checklists, Lifewings Partners also works to educate patients on safety before they even enter the hospital. The company suggests that all consumers do the following: go online to obtain public information on a hospital's safety, talk to their doctors to see what safety standards are in place already, and ask professionals about which facilities tend to have the best safety records. Founder Steve Harden says, "Just because a hospital has a great reputation for cutting-edge medicine doesn't necessarily mean the hospital is the safest place to go for routine procedures." After all, some mistakes are too big & too irrevocable to risk.

READING COMPREHENSION- Comparative Life Expectancy... (A)

Where would you expect the United States to rank in life expectancy, the average number of years a person is expected to live? Near the top in the world, right? Well, surprise -- we rank number 31, after Costa Rica and right before Cuba. World Health Organization data from 2015 indicate that the average life expectancy worldwide is 71 years -- slightly more for females and slightly less for males. In the United States, we beat that average by around eight years. Yet we are several years behind Japan, with a female life expectancy of 86.8 years, or Switzerland, with a male life expectancy of 81.3 years. Even reus, and Malta, do better than we do. There are a number of indicators that lead to our low score among wealthy nations. Those include obesity and its accompanying heart disease and diabetes, car crash deaths, gun violence, and embarrassingly high infant mortality rates. We eat more, drive more, have more guns, and allow poor women to go through pregnancies unattended by regular physicians. There are many other health indicators that separate us from other developed nations, but most correspond to the extreme wealth inequities in the United States. It is too soon to tell whether an increase in insured Americans will help to turn this distasteful data around. Certainly those nations with universal health care have better numbers than we do. Much of our problems has to do with lifestyles that will not be easy to alter. We continue to build living spaces that are not walkable, to work long hours and drive long distances, to overeat -- and we love our Second Amendment right to bear arms. It is difficult to imagine how we will overcome these realities and improve our longevity status.

READING COMPREHENSION- Studying Alzheimer's... (A)

Years of research have proven that Alzheimer's disease, along with other types of dementia, elevates the risk of dying early in the majority of patients. In a recent study performed by the Institute of Public Health at the University of Cambridge, scientist set out to determine exactly how long people were likely to survive following the onset of dementia. Currently, approximately 24 million people throughout the world suffer from the memory loss & orientation confusion that comes with Alzheimer's disease & other forms of dementia. That number appears to double every 20 years, & experts predict that by the year 2040, there will be 81 million people living with some level of the condition. The more researchers & doctors can learn about what causes the problem, as well as how to treat it, the better prepared they will be to handle these millions of future patients. To determine how people's life spans are affected by this medical condition, the scientists studied 13,000 seniors for a period of 14 years. During that time, 438 people developed dementia, the vast majority of whom died. The factors of age, disability, and gender were analyzed to see how they affected longevity as well. Conclusions from the study showed that women tended to live slightly longer than men, averaging 4.6 years from the onset of dementia, as opposed to 4.1 years for men. The patients who were already weak of frail at the onset of the dementia died first, regardless of age. Martial status, living environment, and degree of mental decline, although relevant factors, were not shown to be influential. Researchers from the University of Cambridge hope that this new information will help patients, clinicians, care providers, service providers, policy makers, and others who deal with dementia. The more they known, the better they will be able to respond to this heartbreaking condition.


Kaugnay na mga set ng pag-aaral

Series 66 Chapter 2 Exam Questions

View Set

Derivatives of Exponential and Logarithmic Functions

View Set

CS II Chapter 13 Quiz, CS II Chapter 12 Quiz, CS II Chapter 11 Quiz, CS II Chapter 19 Quiz, 18

View Set